Сохранен 575
https://2ch.hk/spc/res/292138.html
Домены arhivach.top и arhivach.site временно не функционируют! Используйте домен ARHIVACH.XYZ.
24 декабря Архивач восстановлен после серьёзной аварии. К сожалению, значительная часть сохранённых изображений и видео была потеряна. Подробности случившегося. Мы призываем всех неравнодушных помочь нам с восстановлением утраченного контента!

Тред тупых вопросов #50

 Аноним 03/05/16 Втр 23:00:51 #1 №292138 
14623056511160.jpg
14623056511181.jpg
14623056511182.png
14623056511213.png
Тред вопросов о жизни, Вселенной и всем таком.

Спрашиваем то, за что в других местах выдают путёвку в биореактор. Здесь анонимные ученые мирового уровня критически рассмотрят любые гениальные идеи и нарисованные в Paint схемы.


Прошлый тред https://2ch.hk/spc/res/287328.html
Аноним 03/05/16 Втр 23:06:56 #2 №292142 
Так че там, планируются миссии на Каллисто с людьми в обозримом будующем или хоть на бумаге?
Аноним 03/05/16 Втр 23:16:33 #3 №292145 
Какая самая большая масса/радиус космического тела, на котором человек в прыжке может ускориться до второй космической? А до первой?
Аноним 03/05/16 Втр 23:16:52 #4 №292147 
>>292142
Нет, не планируется. Даже на Венеру есть маня-проект, в виде видеоролика, а на Каллисто нет.
Аноним 03/05/16 Втр 23:30:28 #5 №292150 
>>292147
> Даже на Венеру есть маня-проект
Аж загрустил от подобного долбоебизма. Твёрдая земля няшной расцветки, рядом могучий Юп, работают СБ, так нет - подавайте сущий АДЪ, на маня-дирижаблях, в холодильнике поплывём.
Аноним 03/05/16 Втр 23:32:09 #6 №292151 
>>292150
>рядом могучий Юп
>нет - подавайте сущий АДЪ
Вообще-то близкое расположение Юпитера и есть сущий ад. Не говоря уже о расстоянии.
Аноним 03/05/16 Втр 23:33:30 #7 №292152 
>>292145
Диаметр 11 км, масса 1,3910^15 кг, плотность 2000 кг/м³, ускорение свободного падения на поверхности 0,031 м/с², 2-я космическая скорость 5,8 м/с. Диаметр 11 км, масса 1,3910^15 кг, плотность 2000 кг/м³, ускорение свободного падения на поверхности 0,031 м/с², 2-я космическая скорость 5,8 м/с.
Аноним 03/05/16 Втр 23:39:52 #8 №292155 
>>292151
Отнюдь, от Юпа далеко, да и приливной захват же, на обратной стороне очень мало ваших зивертов.
Ну а по расстоянию ближе из нормальных вариантов только гравитационная яма Марса и бездушная Церера.
Аноним 03/05/16 Втр 23:41:42 #9 №292157 
>>292150
Если ты маня-мисси НАСА с дирижоплями на Венеру, то там 100% пиздеж-попил-хуета с красивыми картинками.
>работают СБ
Да, но плохо.
>>292151
Каллисто вне радиационного пояса, вообщет.
Планируют АМС на Европу и Ганимед - а эти-то как раз в радиационном поясе Юпа. Вот там Адъ и Израиль.
Аноним 03/05/16 Втр 23:54:10 #10 №292162 
>>292155
>>292157
А хули там делать, в этих ебенях? Тем более живым хуманам?
Аноним 03/05/16 Втр 23:54:52 #11 №292163 
>>292157
> но плохо
Так и проблемы свободного места нет - тонны тонкоплёночных СБ, в температурных условиях Каллисто не подверженных тепловой деградации это совсем неплохо.
Аноним 03/05/16 Втр 23:57:03 #12 №292165 
>>292162
Будто есть смысл слать человеков на Луну и Марс.
Аноним 04/05/16 Срд 00:00:14 #13 №292167 
>>292165
Ну Марс еще осилить надо, и потом - это планета земной группы, туда-сюда, водичка там была жиденькая, геология, интересно.
А Каллисто вообще хер доедешь.
Аноним 04/05/16 Срд 00:09:11 #14 №292170 
>>292167
> Ну Марс еще осилить надо
Всё к этому идёт.
> и потом - это планета земной группы
Большой оранжевый камушек, без атмосферы(одна сотая нещитова), да и хуй взлетишь с него.
> туда-сюда, водичка там была жиденькая
...да испарилась почти вся.
> геология, интересно.
Там всё остыло считай давно, а у Каллисто аж целый ПОДЛЁДНЫЙ ОКЕАН
Аноним 04/05/16 Срд 00:14:06 #15 №292171 
>>292170
>аж целый ПОДЛЁДНЫЙ ОКЕАН
Тоже мне редкость, по нынешним временам-то. Толку от него - хер докопаешься.
А что там у Марса под поверхностью - еще большой вопрос.
Аноним 04/05/16 Срд 00:36:34 #16 №292173 
14623113949570.jpg
>>292155
>на обратной стороне очень мало ваших зивертов.
Там вроде выходит больше допустимой годовой дозы. ~0.5 Зиверта/год, а это уже не есть гут. Плюс, неэкранизированные вспышки на Солнце, плюс Юпитер - второй по мощности радиоисточник в СС совсем рядом.
Аноним 04/05/16 Срд 01:16:40 #17 №292176 
>>292171
> Тоже мне редкость, по нынешним временам-то. Толку от него - хер докопаешься.
> А что там у Марса под поверхностью - еще большой вопрос.
Один хер, проще научить роботов в ведро, кирку и лопату чем людей слать. Людей всё равно шлют не ради задач, а лишь бы выебнуться.
>>292173
> Там вроде выходит больше допустимой годовой дозы. ~0.5 Зиверта/год, а это уже не есть гут.
Собственной радиации там совсем-совсем мало, Юп не пробивает рентгеном километры Каллисто насквозь. На обратной стороне должно быть норм всё.
> Плюс, неэкранизированные вспышки на Солнце
Магнитное йоба-поле Юпа разве не экранирует? Совсем-совсем?
Аноним 04/05/16 Срд 02:52:04 #18 №292180 
>>292176
У меня нет первоисточников, но цифры, которые я привел, вроде как не только для обращенной к Юпитеру стороне. А спутник (в отличие от, например, Ганимеда) находится дальше ван-аленовых поясов Юпитера, так что солнечный ветер им (Юпитером) не экранируется.
Аноним 04/05/16 Срд 02:58:45 #19 №292181 
Что если облако А, Скажем из водорода, Которое реагирует с облаком Б, скажем кислород, столкнётся с ним?
Аноним 04/05/16 Срд 03:10:11 #20 №292182 
>>292181
Будет облако состоящее из смеси Н2О, ОН, О, О2, Н2 что не такая уж большая редкость.
Аноним 04/05/16 Срд 03:32:47 #21 №292183 
А если облако гремучего газа начнёт скукоживать в звёздообразование, на начальной стадии будет мини звезда, из за горения в прямом смысле?


Что значит следовые количества? Их можно убрать\скрыть?

Бывают ли вещества, которые можно спутать с другим?
Аноним 04/05/16 Срд 11:03:54 #22 №292200 
>>292180
> У меня нет первоисточников
Я уже сам нашел оригинал, там вообще нихуя толком не указанно, тупо 0,01бэр/сут (Как на МКС, меньше чем на Марсе), очень несвязный мурриканский стиль, к сожалению. Ещё нашел план за 2003 год с полётом(человека) на Каллисто к 2040 году.
> А спутник (в отличие от, например, Ганимеда) находится дальше ван-аленовых поясов Юпитера, так что солнечный ветер им (Юпитером) не экранируется.
Не путай тёплое с мягким. Каллисто в 26,3 RJ от Юпа. Ударная волна начинается от 82RJ Должно неплохо резаться.
Аноним 04/05/16 Срд 11:24:05 #23 №292202 
>>292200
Чуток обосрался с переводом величин. 0,01rem=0,1mSv. На Каллисто выходит 10 раз меньше зивертов чем на МКС и Марсе.
Аноним 04/05/16 Срд 13:08:16 #24 №292219 
А магнитное поле экранирует видимый свет?
Аноним 04/05/16 Срд 13:10:44 #25 №292220 
>>292219
Йоба-сильное будет немножко отклонять, не более.
Аноним 04/05/16 Срд 13:12:27 #26 №292222 
>>292220
>будет немножко отклонять
Не будет.
Аноним 04/05/16 Срд 13:13:01 #27 №292223 
>>292222
А почему?
Аноним 04/05/16 Срд 13:13:32 #28 №292224 
>>292223
А почему должен?
Аноним 04/05/16 Срд 13:13:39 #29 №292225 
>>292223
Свет не имеет заряда.
Аноним 04/05/16 Срд 13:38:57 #30 №292227 
А у Вселенной есть магнитное поле?
Аноним 04/05/16 Срд 14:01:48 #31 №292230 
>>292227
В Галактике существует общее магнитное поле. Его силовые линии параллельны галактической плоскости. Изгибаясь, они идут вдоль спиральных ветвей Галактики. Напряженность магнитного поля Галактики около 10-6 э. Оно удерживает диффузные газовые туманности от рассеяния в направлении, перпендикулярном к силовым линиям. Магнитное поле удерживает и космические лучи, порождаемые при вспышках сверхновых звезд. Двигаясь с чудовищными скоростями вокруг силовых линий магнитного поля, частицы космических лучей образуют нечто вроде короны Галактики — обширную сферическую систему, которая невидима.
Аноним 04/05/16 Срд 14:11:52 #32 №292231 
>>292230
И какую роль в этом играет темная материя в гало галактики?
Аноним 04/05/16 Срд 17:56:13 #33 №292297 
>>292231
Никакого. Гало-это просто старые звезды. Физические параметры тёмной материи не определены, кроме как её участия в гравитационном взаимодействии.
Аноним 04/05/16 Срд 18:44:49 #34 №292323 
Купил телескоп, вышел в поле (живу в 20км от города), на что смотреть? На небе нихуя кроме 2-3 более-менее ярких звёзд нет.
Аноним 04/05/16 Срд 18:51:35 #35 №292325 
>>292323
Начни с подглядывания в окна соседей.
Аноним 04/05/16 Срд 18:57:23 #36 №292327 
>>292323
Во-первых, вместо телескопа ты сначала купил бы себе очки, у нас есть телескопотред, в котором сидят владеющие подобной техникой аноны.
Во-вторых, зачем ты вообще его покупал, если не знаешь, нахуй он тебе нужен и что в него смотреть?
В-третьих есть так называемые календари наблюдений, они публикуются регулярно, и доступны на специализированных сайтах. Например на эту неделю:
http://www.astronet.ru/db/msg/1361631
на месяц: http://www.astronet.ru/db/msg/1361156
В-четвертых хорошо, когда известно кому-то еще, кроме тебя, что за инструмент ты купил, таким образом стало бы понятно, на что он способен, и изходя из этого можно было бы что-то посоветовать.
В-пятых скачай Стеллариум, это программа поможет тебе ориентироваться во времени и пространстве.
>На небе нихуя кроме 2-3 более-менее ярких звёзд нет.
Это может значить
1. Все еще очень яркая засветка от населенного пункта.
2. Небо затянуто дымкой.
В обоих случаях - это плохо для наблюдений. Следи внимательно за картами облачности и прогнозом погоды, посмотри на карте светового загрязнения, не сидишь ли ты со своим скопом в какой-нибудь красной зоне или оранжевой зоне.
http://djlorenz.github.io/astronomy/lp2006/overlay/dark.html
Аноним 04/05/16 Срд 19:02:43 #37 №292328 
>>292327
>иcходя
медленно_чиню
Аноним 04/05/16 Срд 20:13:08 #38 №292339 
почему галактики не становятся просто круглыми, почему угловая скорость звёзд одинаковая вокруг центра галактики?
Аноним 04/05/16 Срд 20:17:20 #39 №292340 
14623822408670.gif
Какие шансы что подобные няши полетят в космос?
Аноним 04/05/16 Срд 20:51:02 #40 №292346 
14623842621590.jpg
Если на Венере открыть портал на Марс противоречие между порталами и физикой опустим размером 10 на 10 метров, то сколько атмосферы перетечет? Явно не пополам, т.к. гравитация, размеры и температура. И какие эффекты будут наблюдаться?
Аноним 04/05/16 Срд 20:54:17 #41 №292347 
>>292340
Не нулевые.
Аноним 04/05/16 Срд 21:00:30 #42 №292348 
>>292339
>почему галактики не становятся просто круглыми
В основном потому что вращаются, кроме того, старые эллиптические галактики имеют почти круглую форму, так что галактики вполне себе становятся круглыми иногда.
>>292339
>почему угловая скорость звёзд одинаковая вокруг центра галактики?
Потому что есть проблема скрытой массы и она пока не разрешена окончательно.
Аноним 04/05/16 Срд 21:54:16 #43 №292356 
>>292138 (OP)
Можно ли вывести снаряд из пушки на орбиту если стрелять с Эвереста?
Аноним 04/05/16 Срд 22:02:34 #44 №292359 
>>292356
Нет. Только на суборбиту
Аноним 04/05/16 Срд 22:04:10 #45 №292360 
>>292356
Смотря какая пушка.
Аноним 04/05/16 Срд 22:08:54 #46 №292361 
>>292360
Любая. Чтобы выйти на орбиту нужен корректирующий импульс
Аноним 04/05/16 Срд 22:10:24 #47 №292362 
>>292361
>>292360
>>292359
Ньютон говорил что можно.
Аноним 04/05/16 Срд 22:12:34 #48 №292364 
Напомните название ядерных испытаний когда стальная крышка шахты покинула сраную земляшку со скоростью овер 50 м/с
Аноним 04/05/16 Срд 22:12:52 #49 №292365 
>>292362
Пруф
Аноним 04/05/16 Срд 22:20:08 #50 №292366 
>>292365
https://en.wikipedia.org/wiki/Newton%27s_cannonball
Аноним 04/05/16 Срд 22:22:54 #51 №292367 
В чем соль башни обслуживания на "восточном", если РН из цеха все равно выкатывают в горизонтальном положении?
Если бы в вертикальном как на "канаверале" то еще понятно.
Аноним 04/05/16 Срд 22:26:52 #52 №292368 
>>292367
Что бы работяги не позамерзали нахуй на ветру.
Аноним 04/05/16 Срд 23:26:24 #53 №292394 
Почему к ракете не приделают пушку, чтобы можно было стрельнуть спутником в космосе? https://www.youtube.com/watch?v=bDoh8zQDT38
Аноним 04/05/16 Срд 23:35:38 #54 №292400 
>>292366
Если бы на планете не было бы атмосферы вообще, то через 1 оборот спутник врежется в заднюю сторону пушки.
Аноним 04/05/16 Срд 23:38:55 #55 №292402 
>>292394
Не хотят дорогие радиолампы побить внутри спутника.
Аноним 04/05/16 Срд 23:39:57 #56 №292403 
>>292394
Хотя бы потому что пушка улетит с той же скоростью в противоположном направлении. Помимо этого масса практических и теоретических проблем разной степени очевидности.
Аноним 04/05/16 Срд 23:41:56 #57 №292406 
>>292394
По сути ракета уже пушка выстреливающая горячие газы.
>>292403
Аноним 05/05/16 Чтв 01:29:28 #58 №292429 
14624009683480.jpg
14624009683491.jpg
>>292138 (OP)
Хули Ариель вся в трещинах как и Энцелад? Может там тоже вода под коркой?
Аноним 05/05/16 Чтв 01:37:18 #59 №292431 
>>292429
Почему бы и нет. Вода эвривере
Аноним 05/05/16 Чтв 04:34:20 #60 №292439 
>>292327
Спасибо
Аноним 05/05/16 Чтв 15:44:10 #61 №292472 DELETED
>>292429
Блин, вот мне всегда этот Энцелад нравился. Как будто в пакет завернут
Аноним 05/05/16 Чтв 15:53:41 #62 №292475 
>>292472
Что за патологическая тяга к пакетам у тебя?
Аноним 05/05/16 Чтв 16:04:54 #63 №292479 
>>292429
Трещины и разломы могут имет совершенно разное происхождение. Это вопрос к геологам-планетологам, короч.


Аноним 05/05/16 Чтв 16:14:34 #64 №292483 
>>292475
Он — кот.
Аноним 05/05/16 Чтв 16:54:34 #65 №292493 
>>292162
нихуяшеньки, поэтому никуда никто не полетит, можете скринить. Бешеное воздействие радиации на астронавтов при таких перелетах победить не удалось. Поэтому никакого марса, никакой венеры нем не светит.
Аноним 05/05/16 Чтв 16:56:42 #66 №292495 
>>292165
Луна - рукой подать, на нее послали потому что могли, в качестве эксперимента, один раз и давным-давно, никто туда больше не летает и не полетит.
Марс - тем более никто туда лететь не собирается, это сверхпроект для отсталой недоцивилизации типа человеческой, который нам не по силам - радиация, сверхдолгий перелет, вот это все.
Аноним 05/05/16 Чтв 16:58:28 #67 №292496 
>>292142
миссии с людьми в ближайшие 200 лет не планируются никуда, ибо это супердорого и технически пиздец как сложно. Еще вопросы?
Аноним 05/05/16 Чтв 17:34:51 #68 №292503 
>>292493
>>292495
>>292496
ЛОЛ это десант с пораши или кто-то местный тралирует?
Аноним 05/05/16 Чтв 18:25:08 #69 №292518 
>>292503
А причем тут пораша, если они всех человеков даунгрейдят?
Аноним 05/05/16 Чтв 18:47:47 #70 №292524 
>>292518
Самые тупые и упоротые либо с по, либо с зог
Аноним 05/05/16 Чтв 18:50:20 #71 №292526 
>>292503
Это заговородаун, не обращай внимания, если есть желание - можешь репортить.
Аноним 05/05/16 Чтв 21:37:28 #72 №292557 
>>292475
У меня есть знакомый который без пакета на улицу не выходит. Ложит туда авторучку какую или дезодорант и идет
Аноним 05/05/16 Чтв 21:53:52 #73 №292563 
Какова должна быть сила притяжения, что бы не возможны были космические полеты? Ну то есть ракета бы не набирала первой космической даже.
Аноним 05/05/16 Чтв 21:55:20 #74 №292565 
>>292563
Жить на горизонте событий ЧД.
Аноним 05/05/16 Чтв 22:28:16 #75 №292571 
14624764963250.png
Сап. Наткнулся в рекомендация на такой пост. Я считаю что это хуйня. Високосный год необходим чтобы компенсировать то что продолжительность тропического года около 365.25 суток. А 23 часа 56 минут это сидерический период вращения.
Поправьте если неправ.
Аноним 05/05/16 Чтв 22:33:09 #76 №292572 
>>292571
Там это и написано
Аноним 05/05/16 Чтв 22:36:24 #77 №292573 
>>292571
Если бы речь шла про юлианский календарь, то было бы правдой, но мы живем по григорианскому, так что прав ты.
Аноним 05/05/16 Чтв 22:37:29 #78 №292574 
Где ксп-тред?
Аноним 05/05/16 Чтв 22:38:03 #79 №292575 
>>292574
https://2ch.hk/vg/res/16816980.html
Аноним 05/05/16 Чтв 22:39:22 #80 №292576 
>>292572
Где там написано про тропический год?
Аноним 05/05/16 Чтв 22:48:38 #81 №292579 
14624777185700.jpg
Можно ли создать стелс-шаттл, который не будет видно радарами и оптикой с Земли?
Аноним 05/05/16 Чтв 22:51:00 #82 №292580 
14624778603730.png
Если заднюю часть ракеты заключить в гигантскую сферу, будет ли такая ракета двигаться в космосе? Для внешнего наблюдателя это будет безынерционное движение?
Аноним 05/05/16 Чтв 22:56:15 #83 №292582 
>>292580

Сферу разорвет накапливающимися газами.
Аноним 05/05/16 Чтв 23:00:10 #84 №292584 
>>292582
А если сфера ну очень большая? Хоть пару метров пролетит?
Аноним 05/05/16 Чтв 23:06:29 #85 №292586 
>>292579
Ты его выводить в космос в складках мамкиной юбки собрался?
>который не будет видно радарами и оптикой с Земли?
Очень трудно обеспечить малозаметность такой огромной ебалы как шаттл. То же "Окно" запросто обнаруживает 10 см хуйни на высотах от 120 до 40 000 км.
Аноним 05/05/16 Чтв 23:12:37 #86 №292587 
>>292586
Может специальное покрытие какое с нулевым альбедо заебенить или форму. Или его по затмениям смогут вычислить? Сомневаюсь.
Аноним 05/05/16 Чтв 23:17:48 #87 №292589 
>>292578
>с нулевым альбедо
Даже Vantablack отражает немного света. а учитывая, что он черный, то тепло ты как будешь отводить? Черными радиаторами? Не неси хуйни, короче.
>или форму
Какая форма может быть у шаттла, который можно создать в текущих технологиях, кроме как в самолетной? Он же должен не только в космосе побывать, но еще и вход в атмосферу пережить.
Аноним 05/05/16 Чтв 23:23:21 #88 №292593 
14624798018670.jpg
>>292589
Запускаешь аэродинамический шаттл, а потом разворачиваешь особые угловатые формы. При спуске их можно либо втянуть внутрь, либо сбросить, чтобы сгорели в атмосфере. Примерно как этот щит в новом десу-сексе.

Радиаторы нужны новые - да, какую-нибудь сеть тонких шипов-игл или волос.
Аноним 05/05/16 Чтв 23:28:27 #89 №292594 
>>292593
>какую-нибудь сеть тонких шипов-игл или волос.
Которая будет пиздец отсвечивать.
>а потом разворачиваешь особые угловатые формы
Шаттл и так пиздец тяжелая йоба, ты предлагаешь еще тяжелее его сделать.
И зачем тебе угловатые формы? Нахуй они нужны-то? Превратить шаттл в один сплошной уголковый отражатель?
Аноним 05/05/16 Чтв 23:32:05 #90 №292598 
>>292594
Он же черный будет, нихуя не будет отсвечивать и отражать. Плюс - можно в случае чего ложные цели запустить, которые будут отсвечивать мощнее.
Аноним 05/05/16 Чтв 23:41:16 #91 №292601 
>>292598
Радиаторы ты куда проебал, сколько раз тебе повторить нужно?

>в случае чего ложные цели запустить
На Земле ебланы сидят. Взлетел Шаттл и пропал, а потом бац, ничего не было, вдруг ложные цели повсюду, пропал спутник и вхождение шаттла в атмосферу.
И все такие хором: "ну ок. Не знаем кто, куда делось."
Аноним 05/05/16 Чтв 23:52:31 #92 №292609 
>>292601
Твист - ложные цели и есть нагретые и поглотившие тепло радиаторы
Аноним 05/05/16 Чтв 23:54:44 #93 №292611 
>>292609
>Мандула, оставляющая за собой шлейф не заметна.
Ага. Сайфач где находится подсказать?
Аноним 05/05/16 Чтв 23:57:54 #94 №292612 
>>292611
Можно отстреливать когда тебя никто не видит. США например не могут постоянно видеть МКС, есть определенные мертвые точки.

А тепло нельзя как-нибудь в виде газа выпускать?
Аноним 06/05/16 Птн 00:11:10 #95 №292613 
14624826709710.jpg
14624826709711.jpg
>>292612
>Можно отстреливать когда тебя никто не видит.
А говно это потом просто растворяется по твоему? Шлейф никуда не денется, он так и останется. А по нему отлично все отслеживается. Даже задача облегчается.
>А тепло нельзя как-нибудь в виде газа выпускать?
Можно, че, вози еще газ с собой, что бы шаттл забраться повыше не мог, так его удобнее выслеживать будет.

Что бы ты понимал вот фотки, снятые любителями на любительские телескопы. Ты предлагаешь спрятать эту конскую ебалу от специальных систем слежения.
Аноним 06/05/16 Птн 00:33:17 #96 №292619 
>>292613
Так на этих фото гражданская белая и алюминиевая поебень.
Аноним 06/05/16 Птн 00:40:57 #97 №292622 
14624844573090.jpg
>>292619
Аноним 06/05/16 Птн 00:48:15 #98 №292623 
>>292622
Ты всегда постишь этого ржущего чувака, когда сливаешься, как дно без аргументов?
Ясно.
Аноним 06/05/16 Птн 00:52:51 #99 №292624 
>>292623
Ты задачи найди сначала для своих стелс-шаттлов
Аноним 06/05/16 Птн 00:54:19 #100 №292625 
>>292619
Блядь, шизик ебаный, да можно пердеть газом, выкрасить шаттл в черный цвет, никто не найдет, не отследит и ничего не узнает. Все околоземное пространство забито военными черными и чугуниевыми поебенями, не видимыми и не отслеживаемыми ни одной станцией слежения. Они доверху нагружены ядерными боеголовками и ждут своего часа, что бы спиздив спутники врага закидать его территорию ядреными бомбами.
Все, теперь съеби, пожалуйста туда, откуда ты вылез, невменяемый.
Аноним 06/05/16 Птн 01:12:20 #101 №292626 
>>292625
Представляю, что в свое время какой-нибудь даун также писал про подводные лодки
>Блядь, шизик ебаный, можно затопить корабль и сделать подводную лодку, никто не найдет, не отследит и ничего не узнает. Весь океан забито военными черными и чугуниевыми поебенями, не видимыми и не отслеживаемыми ни одной станцией слежения. Они доверху нагружены ядерными боеголовками и ждут своего часа, что бы подкравшись к врагу закидать его территорию ядреными бомбами. Все, теперь съеби, пожалуйста туда, откуда ты вылез, невменяемый.
Аноним 06/05/16 Птн 05:57:57 #102 №292628 
>>292626
Ээм, нет. Подводные лодки успешно были приняты на вооружение прежде, чем запустили первый ядерный реактор, обосновав тем самым практическую возможность разработки ядерного оружия. На момент появления стратегических ЯО подводный флот тем более не был диковинкой.
sageАноним 06/05/16 Птн 06:50:29 #103 №292630 
>>292624
Задача превращения ядерной триады в ядерный квартет. Ракеты на земле, в воздухе, под водой, и наконец в космосе. Только так можно спать спокойно, зная что глобальный удар не уничтожит все одновременно.

Плюс - ядерные ракеты в космосе гораздо более защищены от террористов. Хуй с два кто слетает на орбиту и отвинтит боеголовку, чтобы потом шантажировать крупный город.
Аноним 06/05/16 Птн 07:42:18 #104 №292634 
А тяжело вообще украсть человека с Земли? Я знаю, что это вопрос уровня /zog/, но все же. Допустим пришельцы только сейчас прилетели в СС и им понадобился человек для экспериментов, но они не хотят палить свое присутствие в системе. Как его выкрасть беспалева?

Вот вы говорите, что пуск ракеты невозможно скрыть. Получается даже если пришельцы под видом метеорита спустятся на Землю, то покинуть Землю незаметно они уже не смогут? То есть если у них ракетные двигатели, а не какая-то анти-грав йоба. Нужно высадиться в место, где мало людей, мало наблюдателей. Значится полюс, скорее всего Южный. Но ведь мы все равно зафиксируем взлет ракетного двигателя с Южного полюса?
Аноним 06/05/16 Птн 08:24:10 #105 №292653 
>>292630
Хотели уже. Не взлетело. Есть конвенция по космосу. Нельзя. Укатывайтесь нахуй, фантазеры ебаные.
Аноним 06/05/16 Птн 09:42:01 #106 №292718 
>>292630
> Плюс - ядерные ракеты в космосе гораздо более защищены от террористов. Хуй с два кто слетает на орбиту и отвинтит боеголовку, чтобы потом шантажировать крупный город.
Не смотри больше голливудских фильмов на ночь
Аноним 06/05/16 Птн 09:44:25 #107 №292720 
>>292718
И когда мне их смотреть? Утром что-ли?
Не пойму чего вы такие серьезные - это же тред для тупых вопросов, а потому нет нужды напоминать мне, что мои вопросы тупые.
Аноним 06/05/16 Птн 09:46:05 #108 №292721 
>>292720
И как часто крадут атомные боеголовки?
Аноним 06/05/16 Птн 09:48:48 #109 №292722 
>>292721
Не думаю, что нам доступны такие данные. Все инциденты с ядерным говном засекречивают.
sageАноним 06/05/16 Птн 09:58:07 #110 №292726 
>>292722
И шантаж городов скрыли? Съеби уже в zog со своим "власти скрывают"
Аноним 06/05/16 Птн 11:37:51 #111 №292763 
>>292634
>Как его выкрасть беспалева?
Взять и похитить бомжа. Никто не заметит.
Аноним 06/05/16 Птн 13:55:18 #112 №292804 
>>292763
Как нам с ним улететь то? Шлейф от перегара будет заметен в поясе Ориона. Кого спиздить то беспалева это не проблема, уже нашли. Ты нам скажи как его отсюда увезти чтоб не спалили.
Аноним 06/05/16 Птн 15:38:17 #113 №292831 
>>292138 (OP)
Можно ли построить космический корабль с парусами для солнечного ветра, который будет плыть по гравитационным волнам?
Аноним 06/05/16 Птн 15:54:54 #114 №292837 
>>292804
<scifi>
телепортация на корабль без приземления
</scifi>
Аноним 06/05/16 Птн 16:05:54 #115 №292841 
Почему пишут, что от слабого магнитного поля у Марса практически отсутсвует атмосфера? У Венеры тоже слабое магнитное поле, однако атмосфера по плотности сравнима с Земной.
Аноним 06/05/16 Птн 16:12:36 #116 №292843 
>>292841
Кто так пишет?
Аноним 06/05/16 Птн 16:21:17 #117 №292851 
>>292841
Так как в основном атмосферу выдувает солнечный ветер, а больше всего режет солнечный ветер именно магнитное поле. На Венере гравитация больше и газы тяжелее, хуй сдуешь, а ещё там тупо больше этой самой атмосферы, дольше сдувается.
Аноним 06/05/16 Птн 16:22:57 #118 №292855 
>>292851
За 4млрд лет так и не сдуло?
Аноним 06/05/16 Птн 16:25:05 #119 №292859 
>>292855
Да, ты правильно меня понял, а ещё было пополнение от вулканической поебени.
Аноним 06/05/16 Птн 16:28:04 #120 №292862 
>>292634
если они смогли прилететь сюда, то их техника позволяет осуществлять телепортацию. Просто телепортнут и все.
Аноним 06/05/16 Птн 16:33:32 #121 №292867 
>>292524
>>292518
>>292526
>>292503
Я здесь, хлопцы. Почему заговородаун? Почему траллирует? потому что порвал ваш сказочный манямирок, где люди летают на пердежной тяге на любые планеты сс? Как раз нет, максимум приземленный кун-нефантазер. Это вы тут собрались на своих свистелках и перделках на марс лететь, лол, чуть ли не дату назначили. А я просто сказал, что в ближайшее время это:
1. Технически уберсложно
2. Практически - бесполезно.
Ну вот зачем, скажите мне лететь на какой-то мертвый камень? Чтобы его поковырять? Тем более отправлять живых людей ради нихуя.
Если ви таки имеете пруфы, что полет на марс или энцелад назначен на такую-то дату - назовите.
Аноним 06/05/16 Птн 16:57:02 #122 №292876 
>>292851
>газы тяжелее
Что на марсе со2 что на венере.
Аноним 06/05/16 Птн 17:20:48 #123 №292884 
>>292876
Вот именно, что на Марсе со2 только и осталось. А все остальное сдуло нахуй.
Аноним 06/05/16 Птн 17:37:52 #124 №292887 
>>292884
Марс дальше и меньше, значит с него должно было дольше сдувать.
Аноним 06/05/16 Птн 17:55:21 #125 №292889 
>>292887
Блядь, ньюфаг ебаный ты задал вопрос и получил на него ответ, нахуй ты споришь? Прошлым летом была в прямом эфире конференция по атмосфере Марса и куда она проебалась, смотрели всем пейсачем. Марс не может удержать атмосферу из лёгких газов, типа Н2, О2 или N2 их сдувает нахуй. Вода в верхних слоях под действием излучения распадается на водород и кислород и их сдувает. Это происходит и на Венере и на Марсе. Но есть два но. 1 на Венере более плотная атмосфера, там больше газа, плюс следы недавней геологической активности, то есть на протяжении долгого времени атмосфера получала подпитку из недр, Марс же давно геологически дохлый. 2 гравитация на Венере выше, так что ей гораздо легче удерживать тяжелые газы. Таким образом и там и там атмосфера состоит из СО2 преимущественно, но по выше описанным причинам на Венере она гораздо толще.
Аноним 06/05/16 Птн 18:04:02 #126 №292890 
Почему если в металлический баллон для газа залить 40 литров гелия, то он не станет легче? Но я не уверен, может и станет.
Но пишут что эти 40 литров поднимут 700 шаров.
Аноним 06/05/16 Птн 18:47:57 #127 №292899 
>>292837
Мы уже приземлились, как съебать? Где у вас тот хуй который стелс технологии продвигал в треде, сюда тащите. Пусть мысль развивает.
Аноним 06/05/16 Птн 18:51:00 #128 №292900 
>>292890
Станет на объём баллона при давлении, равном атмосферному, то есть это как к баллону привязать несколько обычных шариков и мешок с песком в 40 кг.



Аноним 06/05/16 Птн 18:52:47 #129 №292902 
>>292900
Немного не понял, как давление влияет на вес гелия?
Аноним 06/05/16 Птн 18:54:14 #130 №292903 
А если шарик заполнить вакуумом он полетит? Ну если предположить, что шарик способен выдержать внешнее давление без потери формы.
Аноним 06/05/16 Птн 18:55:24 #131 №292904 
>>292862
Всё сломалось, ещё раз повторяем. А с вашими технологиями ни хрена не починить, как съебать уже от вас? Телепортов у нас с роду не было.
Аноним 06/05/16 Птн 19:04:50 #132 №292906 
>>292889
>на протяжении долгого времени атмосфера получала подпитку из недр
На Марсе самый большой вулкан в Солнечной Системе который был активен 200млн лет назад.

На Ио и сейчас идет вулканическая активность, однако никакой подпитки газов атмосфера не получает. У Ганимеда сильное магнитное поле, но атмосферы нет, у Титана слабое поле, но есть плотная атмосфера. Так что это все сказки, что атмосферу Марса сдул солнечный ветер из за слабого магнитного поля.
sageАноним 06/05/16 Птн 19:29:40 #133 №292913 
>>292831
Нет нельзя
sageАноним 06/05/16 Птн 19:30:35 #134 №292914 
>>292904
Если тебе будет легче то я сейчас придумаю бога из машины
Аноним 06/05/16 Птн 19:57:49 #135 №292920 
>>292902
Почему кирпич не плавает?
Аноним 06/05/16 Птн 20:01:18 #136 №292921 
>>292903
Да. Ты только что изобрел вакуумный дирижабль. Про своё изобретение можешь почитать в википедии.
Аноним 06/05/16 Птн 20:02:19 #137 №292923 
>>292899
Я потому и спрашиваю. Может если вы отказываете человечеству в способности сейчас сделать стелс, то объясните как пришельцы смогут в стелс учитывая что у них есть технологии. Телепортация не вариант.
Нужно похищать либо космонавта с орбиты выставив это как несчастный случай, либо ученого с Южного полюса. Там подумают, что он в расщелину упал и пропал.
Может у пришельцев есть тонкий выхлоп ракеты какой-то, который быстро исчезает, но вопрос тем, как скрыться от радаров и ОКА на орбите остается.
Аноним 06/05/16 Птн 20:12:07 #138 №292924 
>>292906
>Упоротый дебил продолжает сравнивать убогие параши с Венерой. Тебе, дауну, третий раз пишу про силу притяжения на Венере.
sageАноним 06/05/16 Птн 20:21:58 #139 №292926 
>>292923
Ты что, парень, совсем тупой? Нужно стартовать с территории Ирана или северной кореи
Аноним 06/05/16 Птн 20:22:48 #140 №292927 
>>292924
Ну и как тогда "убогая параша" - Титан удержиивает свою атмосферу?
sageАноним 06/05/16 Птн 20:48:27 #141 №292938 
>>292927
Потому що он охуенно далеко от солнца - рас, атмосфера из тяжёлых газов - два
Аноним 06/05/16 Птн 21:01:34 #142 №292939 
>>292938
Атмосфера Венеры из CO2, а он тяжелее чем N2 из которого состоит атмосфера Титана.
В далеке от Солнца огромное количество других объектов, но атмосферы сравнимой с Титаном на них нет.
sageАноним 06/05/16 Птн 21:14:57 #143 №292941 
>>292939
Потому что их эволюция пошла по другому пути
Аноним 06/05/16 Птн 21:27:29 #144 №292945 
>>292927
Магнитное поле Сатурна отклоняет СВ. Титан в ебенях.
Аноним 06/05/16 Птн 21:49:34 #145 №292947 
>>292906
Так Титан йоба размером больше Меркурия с 1/7g, да и солнечный ветер рассеивается с расстоянием.
Аноним 06/05/16 Птн 22:04:40 #146 №292949 
>>292927
Убогая параша Титан холодный как губы твоей мамаши, когда она сосет мой хуй, что не сильно способствует избеганию газов из его атмосферы.
Более того, Титан на дохуя процентов состоит из аммиака и его гидратов, который отлично восполняет потери азота. Более того, Титан теряет атмосферу, просто еще не успел ее протерять до конца, по вышеперечисленным причинам.
Аноним 06/05/16 Птн 22:31:14 #147 №292961 
Можно ли КА в космосе раскрутить вокруг двух перпендикулярных осей одновременно, чтобы он так всё время вращался?
Аноним 06/05/16 Птн 22:58:42 #148 №292966 
>>292961
Теоретически можно. Практически что-нибудь (солнечный ветер, астероиды-пылинки и т.д.) все испортит рано или поздно. Да и чистого вращения не будет, возникнет прецессия.
Аноним 06/05/16 Птн 23:04:23 #149 №292969 
>>292966
Я про сферический вакуум, где нет никаких сил, создающих прецессию и солнечного ветра. Это не противоречит каким-нибудь законам сохранения момента, что осей вращения несколько?
Аноним 07/05/16 Суб 00:07:01 #150 №292981 
Как планеты около нейтронных звёзд оказались? Или сформировались, чё за нах? Пульсар-пизда всему же. Взрыв сверхновой и всё такое.
Аноним 07/05/16 Суб 00:11:57 #151 №292982 
>>292981
Сформировались после взрыва сверхновых. Если ты про PSR 1257+12, то хз вообще как это вышло. Возможно был второй компонент, в системе, в последствии проебаный. Но вообще вопрос открытый.
Аноним 07/05/16 Суб 00:20:23 #152 №292988 
>>292982
А из чего они эти планеты не известно? И чё там на них вообще может быть, и кстати там какие условия то на них? Ад и погибель?
Аноним 07/05/16 Суб 00:42:10 #153 №292994 
>>292988
Трудно сказать, зависит от их происхождения. Если это ошметки именно от взрыва сверхновой, то такая планета будет состоять из тяжелых элементов, как минимум углерод и дальше. Если из ошметков второго компонента, то как раз от углерода и ближе к водороду. Условия адские, Чернобыль в момент аварии - курорт и райское место, по сравнению с условиями на этих планетах.
Аноним 07/05/16 Суб 11:13:17 #154 №293032 
>>292904
>Телепортов у нас с роду не было.
Каким же образом вы оказались за тысячи световых лет?
Аноним 07/05/16 Суб 11:33:30 #155 №293034 
Если со спутника на геостационарной орбите спустить верёвку на землю и прикрепить, так, чтобы центра масс остался на геостационарной орбите. Спутник начнёт падать на землю или, натягивая верёвку, пытаться улететь прочь?
Аноним 07/05/16 Суб 13:11:42 #156 №293041 
>>293034
Гугли "космический лифт". Ты его только что изобрёл.
Аноним 07/05/16 Суб 13:26:03 #157 №293042 
>>293032
летаешь с такой с лоренц-фактором 100, за 10 лет 1000 световых делаешь
Аноним 07/05/16 Суб 13:42:49 #158 №293044 
>>293042
До середины войда 10000-е поколение долетит.
Аноним 07/05/16 Суб 14:14:23 #159 №293048 
СМОТРИШЬ СТАРУЮ ПЕРЕДАЧУ ПРО КОСМОС
@
ГОВОРЯТ, ЧТО СТРОИТЕЛЬСТВО МАГЕЛЛАНОВА ТЕЛЕСКОПА ЗАКОНЧАТ В 2015
@
СМОТРИШЬ ВИКИ
@
СТРОИТЕЛЬСТВО НАМЕРЕННЫ ЗАВЕРШИТЬ В 2025
@
ДОСТИГАЕШЬ ТЕМПЕРАТУРЫ ГОРЯЧЕГО ЮПИТЕРА


За что нам это всё?
Аноним 07/05/16 Суб 14:33:42 #160 №293061 
>>293048
Не у всех боссов наса ещё виллы на маями.
Аноним 07/05/16 Суб 15:52:41 #161 №293073 
>>293042
У нас йоба двигатели во много раз (1000) превыщающие скорость света, вам ещё не понять. Все ваши познания о физике и пространстве, как и вселенной в частности ошибочны.
Аноним 07/05/16 Суб 18:26:46 #162 №293097 
14626348067550.jpg
>>293061
>GMT
>NASA
Аноним 07/05/16 Суб 19:06:18 #163 №293102 
14626371785610.jpg
Поясните залетному почему Россия занимает всего 0,6% на рынке космических услуг,если Россия впереди планет ы всей по запускам ракет-носителей?Разве это не самая важная часть в космонавтике?
Аноним 07/05/16 Суб 19:06:29 #164 №293103 
>>293048
>Лунная_база_в_2015.jpg
Запомни, в космической отрасли сейчас все прогнозные сроки всегда переносятся. Поэтому, когда что-то обещают к 2018 — читай будет к 2020, если к 2020 — будет к 2025, а если говорят "после 2030 года" — читай, не будет вообще, потому что это либо маняфантазии, либо концепция к тому времени 100 раз поменяется, и будет что-то совсем иное.
Аноним 07/05/16 Суб 19:17:41 #165 №293104 
Спейсаны, какие на ютубе смотрите каналы о космосе?
Аноним 07/05/16 Суб 19:22:55 #166 №293105 
>>293102
Потому что кроме запусков есть хотя бы создание ПН на заказ. А там уже не выедешь на советском заделе, да и у совка с этим всегда были проблемы - спутники жили гораздо меньше западных. Причин дохуя, от элементной базы до низкой культуры производства.
Аноним 07/05/16 Суб 19:35:07 #167 №293107 
14626389079400.jpg
>>293102
>впереди планеты всей по запускам
>Разве это не самая важная часть в космонавтике?
Да, настолько же важная и значимая, как бизнес таксистов в общей экономике мегаполиса. Запуски — всего лишь средство, цель же — услуги связи, геологоразведка, картография, погода, навигация. Это все обеспечивают спутники и их начинка, с которыми у россиюшки сейчас полный провал, если говорить о коммерческом секторе. Да, по военным заказам что-то еще клепают, и то оно работает криво — то солнечные панели не раскрываются, то системы стабилизации на работают, то еще что-то не так. После ввода санкций и перехода на "импортозамещение" дела еще хуже. Донедавна вообще спутники-шпионы фотографировали на пленку, капсулы с которой затем по тайге искали с собаками, как диды. Из-за глубокого отставания в элементной базе отечественные спутники имеют ресурс работы в несколько раз меньше зарубежных того же класса, поэтому никто в здравом уме заказывать их для коммерческих целей не будет, кроме военных, у которых нет выбора. Единственное, в чем сейчас у России есть безусловное преимущество — это возможность пилотируемых запусков, и то это очень временно, поскольку уже через 2 года полетят масковский Дракон и боинговский Старлайнер, а так даже на рынке запусков Роскосмос теснят уже сейчас.
Аноним 07/05/16 Суб 19:47:53 #168 №293111 
>>293104
Никакие, там нихуя нет интересного, надо книги читать.
Аноним 07/05/16 Суб 19:50:22 #169 №293112 
>>293107
Электронную начинку с запада вроде как начали или пытаются покупать через пиздоглазых и индусов
Аноним 07/05/16 Суб 19:50:50 #170 №293113 
>>293111
Тогда хоть книжек годных по астрономии/астрофизике посоветуй.
Аноним 07/05/16 Суб 19:53:10 #171 №293115 
>>293113
А не, пизжу: смотрю канал Постнауки.
Аноним 07/05/16 Суб 20:13:55 #172 №293121 
>>293112
Китайские братушки вроде как прокинули таких начинателей вокруг хуя, и как красиво! На глазах у всего мира разыграли все, как по нотам. Сначала взяли деньги, купили нужные компоненты, а потом сказали "яяй, мы не можем отдать, санкции, да". Деньги, естественно, не вернули.
Аноним 07/05/16 Суб 21:49:15 #173 №293135 
Сука, заебали.
Столько инженерных задач вокруг. Столько хуйни планетологической.
Нет же, каждый полуграмотный долбоёб с умным видом вбрасывает вопросы про св. Струны и пердолится в свою ЧД. И такие же полуграмотные ему отвечают, причем полнейшую ахинею, зачастую.
Создайте себе астрофизики тред, и фантазируйте там до схлопывания. Всю нить засрали.
Аноним 07/05/16 Суб 22:16:16 #174 №293142 
14626485767100.png
>>293135
>Столько инженерных задач вокруг. Столько хуйни планетологической.
Аноним 07/05/16 Суб 22:33:44 #175 №293145 
>>293042
>летаешь с такой с лоренц-фактором 100
Ага, а потом при подлете включаешь фотонные двигатели для торможения, который будут сиять на полнеба, как рождественская елка. Теплый прием и хлеб-соль-плутоний от встречающей делегации гарантированы.
Так что либо йоба-технологии и непалевный телепорт, либо о твоем подлете узнают минимум лет за 20.
Аноним 08/05/16 Вск 00:12:36 #176 №293162 
https://youtube.com/watch?v=lF5R_qEbTsA
Звук получается тоже с задержкой идёт? Как вообще могли поймать звук чёрной дыры если даже свет не может её покинуть?
Аноним 08/05/16 Вск 00:21:37 #177 №293167 
>>293162
>Как вообще могли поймать звук чёрной дыры если даже свет не может её покинуть?
Это радиоизлучение, естественно не самой ЧД, а вещества, которое разогревается в аккреционном диске и излучает, в том числе и в радиодиапазоне.
Аноним 08/05/16 Вск 02:45:45 #178 №293195 
Может кто-нибудь пояснить за радиационную обстановку в окрестностях Меркурия? С одной стороны там должен быть значительно выше уровень солнечной радиации, с другой стороны близость Солнца должна снижать уровень галактического высокоэнергетического говна, плюс ещё меркурианское магнитное поле.
Аноним 08/05/16 Вск 03:10:34 #179 №293197 
>>293195
Обстановка адовая. Меркурий, конечно, имеет магнитное поле, однако оно гораздо слабее того же земного, так что из-за близости к Солнцу оно вполне пробивается сильными коронарными выбросами.
>в окрестностях Меркурия?
Это самое магнитное поле размером примерно в диаметр Земли, так что окрестности не сильно им защищены.
Отсутствие атмосферы оставляет меркурий без защиты от прочих видов излучения. Так что принимай поверхность дозу: , магнитное поле гамма и рентген не останавливает. УФ тоже не блокируется.
Кстати, из-за этого самого солнечного излучения на Меркурии довольно интересные процессы происходят:
http://messenger.jhuapl.edu/news_room/presscon_multi6.html

>близость Солнца должна снижать уровень галактического высокоэнергетического говна
Близость солнца никак не снижает уровень высокоэнергетического говна. Прилетело так прилетело. Земляшку атмосфера оберегает, а Меркурий оберегать нечему.
Аноним 08/05/16 Вск 03:28:53 #180 №293198 
14626673338090.jpg
Насколько я понял, с т.з. ракеты двигающейся с 1 жи, скорость не ограничена с, в той степени что 1 секунда будет длится дольше. Так будет ли с той т.з. равноускоренное передвижение или там будут вариативное ускорение?
Аноним 08/05/16 Вск 03:39:15 #181 №293199 
>>293197
>Обстановка адовая.
Мне бы цифры, хотя бы приблизительные.
>Это самое магнитное поле размером примерно в диаметр Земли, так что окрестности не сильно им защищены.
Тем не менее MESSENGER там летал, должен же был получить данные.
>магнитное поле гамма и рентген не останавливает. УФ тоже не блокируется.
Меня больше интересует проникающая радиация.
>Близость солнца никак не снижает уровень высокоэнергетического говна.
Почему? Пики солнечной активности совпадают с минимумами галактического излучения и наоборот. Логично что и по приближению к самому Солнцу уровень должен падать. Разве нет?
Аноним 08/05/16 Вск 03:48:00 #182 №293200 
>>293199
Так то Солнце куда более мощный источник высокоэнергетического говна, чем галактическое и внег-е на многие порядки. Может быть на орбите Нептуна и не так, но даже на сраных 150 млн. км это полностью так.
Аноним 08/05/16 Вск 06:24:14 #183 №293205 
>>293162
Все эти "звуки Юпитера", "звуки галактики Андромеды" и прочая есть не что иное, как переложение излучения в радиодиапазоне на слышимый человеком звуковой диапазон. Причем довольно вольное переложение, просто функция колебаний переносится в каком-нибудь звуковом редакторе на некую произвольную тональность. Причем если недостаточно "торкает", то можно накрутить эффектиков. Никто ведь не будет лезть в радиограммы и сверять колебания. Какая-либо научная польза или прикладная ценность в этих записях не обитает.
Аноним 08/05/16 Вск 06:36:05 #184 №293206 
Какие процессы в Солнце отвечают за испускание радиоволон, а какие - за рентген и гамма излучение?
Аноним 08/05/16 Вск 12:31:42 #185 №293234 
>>293205
>Какая-либо научная польза или прикладная ценность в этих записях не обитает
А теперь пруф этого на авторитарном источнике.
Аноним 08/05/16 Вск 12:36:13 #186 №293236 
>>293234
Пруф на отсутствие? Ты ебнулся? Эти т.н. звуки космоса и правда делают потехи ради.
Аноним 08/05/16 Вск 12:38:28 #187 №293237 
14627003081800.jpg
>>293234
>авторитарном источнике.
Это в газете Правда за 51-й год что ли?
Аноним 08/05/16 Вск 12:39:11 #188 №293238 
>>293236
>звуки космоса и правда делают потехи ради
Тогда пруфс?
Аноним 08/05/16 Вск 13:18:47 #189 №293246 
14627027271560.gif
14627027271561.gif
14627027271572.gif
>>293238
Блядь, какой тебе пруфс, ебанутый? Что ученые не слушают ухом черные дыры, а ковыряются в унылых графиках, которые получают с телескопов?

Автор конкретно этого переложения Эдвард Морган из MIT. Он наблюдатель (Instrument scientist). Тот мужик, который проводит наблюдения, то есть на деле глядит, что бы инженеры все правильно сделали, согласно графику наблюдений, который ему дают. Работает он с RXTE. Это пиздец какая скучная работа. Вот он от нехуй делать взял кусок наблюдений в рентгеновском диапазоне и перегнал в звук.
Вот список его публикаций, найди там упоминание звуков GRS 1915+105.
http://space.mit.edu/home/ehm/
Аноним 08/05/16 Вск 15:11:18 #190 №293254 
>>293234
Какие тебе пруфы? Человек своими ухами конкретно вообще довольно плохо оперирует в научном смысле. В теории музыки - да, но в астрофизике... Никто ведь не определяет скорость и направление ветра на Марсе, упорно прослушивая запись завываний этого самого ветра. Наблюдение ведется по графикам и спутниковым снимкам. А тебе тут пруфы подай, что никто по синтетическим звукам черной дыры ее звездное прошлое не изучает? Я заикнулся про прикладную ценность только потому, что человек, задававший вопросты ли это был? спрашивал сущий примитив про звук из космоса, как так типа. Это выдает очень низкий уровень познания астрофизикине как что-то плохое даже по меркам научпопа. Вот я и вставил для проформы утверждение, которое каждому завсегдатаю здесь очевидно. Но ты, вероятно, закален парашей, так что впредь пиши в своих постах, что "безпруфных кукареков" тебе не надо, и я тогда буду молча обходить твои посты.
Аноним 08/05/16 Вск 19:01:15 #191 №293280 
>>292339
> галактики не становятся просто круглыми
большинство галактик это обрывки разных галактик, например наша оторвала кусок от малого магелланового облака в своё время. И через время от нас оторвёт кусок андромеда, мбо массивнее
Аноним 08/05/16 Вск 19:03:41 #192 №293281 
>>293206
одинаковые - нуклеосинтез
Аноним 08/05/16 Вск 19:09:28 #193 №293283 
>>293197
> магнитное поле гамма и рентген не останавливает
у солнца рентгена и особенно гаммы очень мало, из говен можно разве что нейтронов подцепить, но они дальше 1 млн км не летят.
> УФ тоже не блокируется
ваще похуй, накройся фольгой из люминия.

все траблы с солнцем - это онли альфа-бета частицы разных энергий во время вспышек, если правильно прогнозировать можно прятаться под почву, и на меркурии еще ИК пичот неплохо.
Аноним 08/05/16 Вск 19:12:20 #194 №293284 
>>293198
с тз земляхи будет замедление ускорения по мере приближения к скорости света, с тз ракеты по достижению скорости света ракета долетит мгновенно куда угодно, время перестанет течь. в теории.
Аноним 09/05/16 Пнд 01:29:16 #195 №293361 
>>293283
>все траблы с солнцем - это онли альфа-бета частицы разных энергий во время вспышек
Разве СКЛ это только продукт вспышек? Мне казалось что Солнце чадит в непрерывном режиме.
Аноним 09/05/16 Пнд 21:32:03 #196 №293465 
14628187236380.jpg
Проект корабля с максимальной скоростью 1000км/с.
Полет к Юпитеру с серией гравитационных маневров, сближение с Солнцем до нескольких млн км, облет Солнца с ростом скорости до 400 км/с. Затем открывается солнечный парус и солнечные батареи. На таком расстоянии эффективность их огромна. После вышеописанных действий путь до Плутона займет три месяца.
Аноним 09/05/16 Пнд 21:35:49 #197 №293466 
>>293465
> нескольких млн. км.
Он из анобтаниума?
А назад как?
Аноним 09/05/16 Пнд 21:37:47 #198 №293468 
>>293466
Щит, как в Пекле. Под углом, чтобы уменьшить сопротивление ветра и излучения. Этот же щит после облета используется в качестве паруса.
Назад никак, это автоматическая станция.
Аноним 09/05/16 Пнд 21:42:26 #199 №293470 
>>293468
Слишком дорого/говно/моча
Аноним 09/05/16 Пнд 21:53:14 #200 №293473 
>>293465
А к Солнцу за счёт чего полетит сия ебала? Силой молитвы?
Аноним 09/05/16 Пнд 22:00:34 #201 №293475 
>>293473
Ну эта... Щит как в пекле, движки как в инстелларе.... А потом через червовоточинку - фьють!!!
Да это залетный из sf, они тут бывают.
Аноним 09/05/16 Пнд 22:01:44 #202 №293476 
14628205043660.png
14628205043811.png
14628205043922.png
14628205044063.png
>>292138 (OP)
У меня тупой вопрос про неудачные запуски и полезный груз. Мне только кажется, что в неудачных запусках участвует только неживой полезный груз вроде жрачки и аппаратуры для МКС, а ракеты с космонавтами всегда долетают? Или я ошибаюсь и космонавты иногда горят. С примерами сгоревших космонавтов, если можно. В наше время, а не из 80х.
Аноним 09/05/16 Пнд 22:22:00 #203 №293481 
>>293476
Последний крайний раз погибали люди на Колумбии в 2003. Сейчас не гибнут потому что пускают на ракетах, которые почти без изменений с 1970х летают (аналоговые СУ, смешные давления в камерах сгорания и т.д.). Людей на новые ракеты не пересаживают до тех пор, пока они статистику не налетают/ну будут одобрены для пилотируемых полётов.
Аноним 09/05/16 Пнд 22:22:22 #204 №293482 
>>293476
> С примерами сгоревших космонавтов, если можно. В наше время, а не из 80х.
Нет таких, Шаттлы не считаются.
Аноним 09/05/16 Пнд 22:27:34 #205 №293485 
>>293482
Хуя себе, семь фрагов за раз не считается?
Аноним 09/05/16 Пнд 22:51:08 #206 №293498 
Вот что хотел узнать, я где-то читал или гифку видел, что планеты у удалённых звёзд вычисляют так: планета проходит между звездой и аппаратом, и гравитация планеты искажает свет так, что он становится в 4+ раз ярче - это правда? Вроде да.

А почему такой эффект не юзают для получения доп. солнечной энергии?
Аноним 09/05/16 Пнд 23:11:22 #207 №293515 
>>293498
Не, ты написал хуйню. Полнейшую хуету просто. Скорее всего виноват кривой русек.
А основных метода два: первый - планета перекрывает звезду и делает её чуть менее яркой, но планеты редко так располагаются, другой вариант - планета массивная йоба, очень близко к звезде и быстро движется, вследствии чего звезда чуть смещается синхронно с вращением планеты, но лёгкие планеты так хуй найдёшь. Ещё есть вариант тупо визуально найти, но это не так тупо и просто как кажется.
Аноним 09/05/16 Пнд 23:16:52 #208 №293516 
>>293515
>Ещё есть вариант тупо визуально найти, но это не так тупо и просто как кажется.
Визуально планету у звезды невозможно увидеть из-за того, что планета не может отражать свет с такой силой, чтобы звезда не "затмевала" её. Это также, как различить спутники Юпитера без инструментария (пример хуевый, но подходит).
Аноним 09/05/16 Пнд 23:18:54 #209 №293517 
>>293473
К Солнцу будет полет после маневров вокруг Юпитера и других планет. Да, сложно и дорого, но это единственный нефантастический вариант разогнать корабль до таких скоростей. Движок тут самый обычный химический на старте, сборка на орбите, потом снова химия и ионники.
Аноним 09/05/16 Пнд 23:19:12 #210 №293518 
>>293515
Я не помню где, но видел точно гифку с объяснением. Суть в том, что есть какой-то эффект, что гравитация искажает вид, и смотришь на 1 свезду, а кажется что их 4. И я читал что так планеты у звёзд находят.
Аноним 09/05/16 Пнд 23:22:12 #211 №293519 
>>292483
>Он — кот.
Или карп
>>292630
Норкоман, ты еще с луны предложи ебашить рокетами. Ничего что с орбиты ракета будет лететь минут 40 до цели и к моменту подлета уже все разъбенят и без этой рокеты? Не говоря уже о том как ты топливо хранить на орбите будешь.
Аноним 09/05/16 Пнд 23:22:56 #212 №293520 
14628253762990.png
>>293518
Либо давай источник, либо это бред. Гравитация искажает вид, например от края Солнца свет может пройти на 2`` "за ним", но из этого никак не может искусственно увеличиваться яркость.
Аноним 09/05/16 Пнд 23:26:03 #213 №293521 
>>293520
Говорю что забыл название, суть в том, что на пике 1 объект за...
Вспомнил - крест Эейнштейна или гравитационная линза. ВОТ ПРО ЧТО я говорил, и как искали планеты.
Аноним 09/05/16 Пнд 23:30:10 #214 №293523 
14628258106270.jpg
>>293521
Я бля понял про что ты. Этот метод не может использоваться с системой звезда-планета из-за того, что планета не искривляет пространство настолько, чтобы мы получили 4 звезды. Для галактик это работает (пикча с вики).
Аноним 09/05/16 Пнд 23:56:52 #215 №293534 
>>293523
А может в центре нет никакой чёрной дыры, и это просто линза?
Аноним 09/05/16 Пнд 23:59:24 #216 №293535 
>>292138 (OP)
> 14623056511182.png
Что за пончик? Ракетостроители-любители балуются, что ли?
Аноним 10/05/16 Втр 00:25:12 #217 №293536 
>>293476
Для людей проверенные ракеты используют и всё тщательно проверяют, а грузов особо не парятся.
Аноним 10/05/16 Втр 00:41:56 #218 №293537 
До сих пор грустно от того, что проебали mars climate orbiter. Всегда интересовался атмосферой Марса, климатом, молнии в пыльной буре, снег, туман, там ведь не только пыль и углекислота. А эти пидоры со своими милями и фунтами все проебали.
Аноним 10/05/16 Втр 01:34:30 #219 №293556 
>>293516
> Визуально планету у звезды невозможно увидеть из-за того, что планета не может отражать свет с такой силой, чтобы звезда не "затмевала" её.
Всё упирается в альбедо, сенсоры, разрешающую способность и алгоритмы. Прецендеты есть, но пока оче сложно.
> Это также, как различить спутники Юпитера без инструментария (пример хуевый, но подходит).
Пример действительно хуёвый. Технически это таки возможно: светочувствительность и разрешающая способность позволяет. История располагает малоизвестным случаем оче зоркого китайца, который таки смог.
Аноним 10/05/16 Втр 15:25:22 #220 №293622 
14628831229490.png
sageАноним 10/05/16 Втр 20:41:58 #221 №293684 
x \* y

проверка возможности искейпить значки умножения, плиз игнор
Аноним 10/05/16 Втр 20:46:46 #222 №293686 
>>293684
С одной звёздочкой/знаком умножения ничего не случится. Макаба их хавает в количестве двух и больше.
Аноним 10/05/16 Втр 21:14:00 #223 №293693 
>>293622
Старо же
Аноним 11/05/16 Срд 02:21:37 #224 №293776 
Можно ли разогнать КА до высоких скоростей (100км/с и более) на химическом двигателе со скоростью истечения 4 км с если топливо условно бесконечное?
Вообще вопрос - может ли двигатель разогнать корабль быстрее скорости истечения его рабочего тела, будет ли создаваться тяга или топливо будет улетать впустую?
Аноним 11/05/16 Срд 07:14:20 #225 №293778 
чому спейсач огурцачом зовут, а анонов-огурцами?
Аноним 11/05/16 Срд 07:18:55 #226 №293779 
>>293776
можно, может
>>293778
KSP, внешний вид кербанавтов
Аноним 11/05/16 Срд 10:52:02 #227 №293794 
Спейсаны, щас смотрю на ютабе лекцию Владимира Сурдина из МГУ интересно вещает. Дык вот какой вопрос у меня возник, а на Луне допустим звуки какие ни будь есть? или там тишина аки на погосте. А бля, чёт пока писал по ходу сам допёр, там же вакуум, значит и волнам не где звуковым быть, верно я допёр? И типо что получается, тихо будет даже когда ракета взлетает? хуйня какая то. Поясните ка мне плиз.
Аноним 11/05/16 Срд 10:54:59 #228 №293797 
>>293794
И вот ещё в догонку вопрос. Какого хрена нам Луна иногда кажется пиздецки большой на небе и разных цветов. Он чёт на эту тему ни чего не ответил, грит науке не известно, что за дела ещё такие, на двоще должны знать.
Аноним 11/05/16 Срд 10:59:08 #229 №293799 
14629535488250.png
>>293797
Вот пик с Луной собственно
Аноним 11/05/16 Срд 11:53:33 #230 №293804 
>>293778
Ксп-треды уже все
Аноним 11/05/16 Срд 11:54:30 #231 №293805 
>>293799
Такой луны не бывает
Аноним 11/05/16 Срд 14:15:03 #232 №293821 
>>293805
Бывает, если снимать с огромным фокусным расстоянием.
>>293794
Земля дрожать будет. В первые секунду-две точно, потом давление газов на стартовый стол будет очень маленькое.
а вообще вкинули бы картинку с распределением давления в факеле MVac какого-нибудь, самому интересно
>>293797
Луна на небе большая, когда она низко над горизонтом, цвет разный тоже из-за атмосферы.
https://youtu.be/Y2gTSjoEExc
видео стронгли релейтед
Аноним 11/05/16 Срд 19:54:46 #233 №293852 
Спейсач, у меня не вопрос, а скорее запрос.
Посоветуй научно-популярных каналов на ютубе про космос. Желательно русскоязычных. В английский могу, но родной как-то приятнее.
Аноним 11/05/16 Срд 21:12:14 #234 №293862 
Почему метеориты не уничтожают спутники? И как ракеты взлетают, если на орбите куча мусора?
Аноним 11/05/16 Срд 22:19:52 #235 №293874 
>>293862
Потому что пространство большое, а спутники, ракеты метеоры и метеориты - маленькие. Теория большого неба во все поля.
Аноним 11/05/16 Срд 23:02:47 #236 №293876 
>>293874
Метеоритов тысячи за сутки влетает, а спутники невредимы на орбите десятилетиями.
Аноним 11/05/16 Срд 23:42:31 #237 №293878 
>>293876
Ты случайно не тот тупорылый уебан, который после каждого ответа на свой тупорылый вопрос начинает тупорыло спорить, мол "так не может быть, вы мне все врете и так далее"?
Аноним 12/05/16 Чтв 00:01:36 #238 №293880 
>>293878
Но так не может быть. Спутник всё равно что муха летающая под дождём.
Аноним 12/05/16 Чтв 00:05:24 #239 №293881 
>>293880
Там особый генератор защитного поля ставят.
Аноним 12/05/16 Чтв 02:01:50 #240 №293891 
>>293481
>пускают на ракетах, которые почти без изменений с 1970х летают (аналоговые СУ, смешные давления в камерах сгорания и т.д.).
Вся суть полупроводниковой электроники и современных учьоных с их реферативными диссертациями.
Аноним 12/05/16 Чтв 08:47:42 #241 №293900 
Можно ли залабать на гитаре в открытом космосе? :)
Аноним 12/05/16 Чтв 08:56:14 #242 №293901 
>>293900
Можно. Только никто не услышит.
Аноним 12/05/16 Чтв 09:27:55 #243 №293903 
>>293900
Можно, но слышно будет только тебе и звук будет совсем другой, т.к. гитара, как и все подобные инструменты, рассчитана на колебания воздуха внутри неё самой.
Аноним 12/05/16 Чтв 09:49:40 #244 №293904 
>>293903
>слышно будет только тебе и звук будет совсем другой
Хмм, а вот тут поподробней. И помедленней пожалуйста, я записываю.
Аноним 12/05/16 Чтв 10:50:47 #245 №293907 
ня
Аноним 12/05/16 Чтв 12:34:23 #246 №293913 
14630456636030.jpg
>>293903
Как ты собрался в таких перчатках играть?
Аноним 12/05/16 Чтв 13:06:08 #247 №293914 
>>293913
Как Майор Том.
Аноним 12/05/16 Чтв 13:11:33 #248 №293918 
14630478940120.jpg
>>293900
Можно, но только на электро. Если с правой рукой более-менее можно выкрутится - плектор там, то, се. То с левой довольно кисло. Нужно спецперчатки мутить. Полноценно играть не выйдет, но слайдом можно погонять вполне прилично.
...был такой мужик, Джанго Рейнхард - у него на левой руке было только ДВА пальца. А играл так, что ребятки с полной клешней нервно курили за углом.
Аноним 12/05/16 Чтв 16:05:45 #249 №293926 
Какой профит от пилотируемых полетов в космос?
Что может сделать человек чего не сможет сделать робот?
В чем профит обитаемых орбитальных станций? 20 лет МКС НИЧЕГО практически человечеству не дали.
Какие перспективы в плане дальнейшего освоения космоса? Ждать ли больше орбитальных станций и баз на других планетах или нет?
Аноним 12/05/16 Чтв 17:34:41 #250 №293934 
>>293926
Никаких. На земле слишком ахуенно, зачем куда-то лететь?
Аноним 12/05/16 Чтв 18:19:38 #251 №293937 
>>293934
Думаю нам нужно создать специальный файл с ответами на популярные вопросы и где-нибудь yf pastebin его держать.
Ты это, не юродствуй, тред не для шуточек
Аноним 12/05/16 Чтв 18:23:25 #252 №293938 
Что будет если высунуть хуй из скафандра в космос и поссать?
Аноним 12/05/16 Чтв 18:23:30 #253 №293939 
>>293926
>Какой профит от пилотируемых полетов в космос?
Что бы мы сами охуели,как мы можем.
На деле профит есть, дохуя технологий пришло из космической отрасли. Кстати, не так давно Спиноф2016 вышел. Полюс архив за прошлые года.
https://spinoff.nasa.gov/Spinoff2016/index.html
Аноним 12/05/16 Чтв 20:16:38 #254 №293952 
Спейсаны, я тут читал, что планировали на МКС доставить ещё один бытовой модуль на основе союза с нормальным душем/сауной как на МИРе. Это дело загнулось что ли?
sageАноним 12/05/16 Чтв 20:17:55 #255 №293953 
>>293904
Как глухие музыканты играют гугли.
Аноним 12/05/16 Чтв 21:18:51 #256 №293957 
>>293534
Ага, блядь, линза размером с четыре галактики.
Аноним 12/05/16 Чтв 21:32:12 #257 №293958 
>>293534
Из-за гравитации ЧД происходит такой эффект. Крест - очень яркий пример этому эффекту. Хочешь сказать повсюду такие линзы?
Аноним 12/05/16 Чтв 22:05:16 #258 №293964 
14630799168130.png
>>293498
Не слушай этих мудаков, такой метод действительно есть, называется гравитационное микролинзирование. Если при линзировании, вызванном достаточно массивными объектами (галактики, массивные черные дыры) видно смещение и искажение фонового изображения, как в кресте Эйнштейна, то при линзировании объектом малой массы, вроде планеты, картинка не меняется, но свет звезды, перед которой проходит планета, усиливается, причем четко по известному графику, что позволяет отличать это от просто переменных звезд. Сейчас таким методом открыто 16 подтвержденных экзопланет и еще больше ожидают подтверждения. Все это описано в вики https://en.wikipedia.org/wiki/Gravitational_microlensing
Аноним 12/05/16 Чтв 22:13:13 #259 №293967 
>>293498
Вдогонку
>он становится в 4+ раз ярче
Не обязательно именно в 4, это зависит от массы линзирующего тела и расстояния до него.
>почему такой эффект не юзают для получения доп. солнечной энергии
Потому что для этого пришлось бы между Землей и Солнцем поместить тело, достаточно массивное, чтоб заметно отклонять его лучи и при этом достаточно мелкое, чтоб их не перекрывать. Единственное, что удовлетворяет обоим этим условиям это черная дыра, причем ее масса должна была бы в сотни раз превышать солнечную. Даже если бы какая-то сверхцивилизация могла это сделать, это был бы очень тупой вариант, поскольку такая дыра распидорасила бы всю систему и в конечном итоге уничтожила бы солнце.
Аноним 12/05/16 Чтв 22:26:11 #260 №293972 
>>293967
Так чёрных дыр не сущевствует
Аноним 12/05/16 Чтв 22:43:40 #261 №293974 
>>293972
А что тогда в центре галактик? А квазары, разве это не черные дыры в фазе активного поглощения вещества?
Аноним 12/05/16 Чтв 22:49:35 #262 №293975 
14630825755310.jpg
>>293852
бамп вопросу
Аноним 12/05/16 Чтв 22:54:30 #263 №293976 
>>293852
Канал Роскосмоса жи.
Аноним 12/05/16 Чтв 22:57:37 #264 №293978 
>>293976
и действительно. Большое спасибо
Аноним 12/05/16 Чтв 23:09:15 #265 №293980 
>>293974
> А что тогда в центре галактик?
Ничего
> А квазары, разве это не черные дыры в фазе активного поглощения вещества?
Нет
Аноним 12/05/16 Чтв 23:15:18 #266 №293982 
>>293980
Тупых ответов тред у нас в бэ
Аноним 12/05/16 Чтв 23:15:29 #267 №293983 
>>293361
Частицы имеют массу, их надо запердолить на вторую космическую относительно солнца сначала, чтобы чадеть ими. Солнце плюётся ядрами атома гелия - альфа частицами, и нейтронами, которые через 15 мин свободного полёта разлетаются на протон и электрон.
Аноним 12/05/16 Чтв 23:16:44 #268 №293984 
14630842047240.gif
14630842047371.gif
>>293980
>Ничего
Нихуевое такое ничего там жонглирует звездами.
Завязывай с толстотой.
Аноним 12/05/16 Чтв 23:17:05 #269 №293986 
>>293517
тормозить возле плутона как? ебалом об поверхность?
Аноним 12/05/16 Чтв 23:17:54 #270 №293987 
>>293982
>>293984
Мне на форчане сказали так
Аноним 12/05/16 Чтв 23:18:32 #271 №293989 
>>293986
Ионниками
Аноним 12/05/16 Чтв 23:18:36 #272 №293990 
>>293987
>на форчане сказали
Ты бы еще к бабкам на рынок сходил, или в церковь.
Аноним 12/05/16 Чтв 23:22:01 #273 №293993 
>>293989
1000км/с? заебёшься
Аноним 12/05/16 Чтв 23:25:29 #274 №293994 
>>293984
В конце гифки звезда пролетает с ебанутой скоростью с долгопериодической орбиты и улетает в ебеня. Это фейк. Как анон, который моделировал движение, могу утверждать что такая скорость возможна только на гиперболической орбите, у которой ветки разогнуты куда сильнее, а там изображена эллиптическая
Аноним 12/05/16 Чтв 23:28:32 #275 №293995 
>>293989
Алсо у ионников дельта ограничена, хоть они и круче реактивных но не до беспредела жи. Просто нельзя взять столько ксенона, чтобы.
Аноним 12/05/16 Чтв 23:29:24 #276 №293996 
>>293994
>В конце гифки звезда пролетает
Какой из двух гифок? Какая именно звезда?
>Это фейк.
Власти скрывают? Заговор ученых?
Аноним 12/05/16 Чтв 23:34:20 #277 №293998 
>>293996
Первая, где-то 1997 год. Там скорость должна быть не более 1.44 от круговой, иначе улетает и не возвращается, а она больше раз в 10. С такой скоростью она должна была пролетать мимо почти по прямой
Аноним 12/05/16 Чтв 23:37:55 #278 №294000 
>>293998
Тоже не понял этот момент.
Аноним 12/05/16 Чтв 23:54:10 #279 №294004 
14630864502180.png
Что будет если выйти в космос без скафандра? Сразу разорвет из за давления? Обморозишься? Облучишься? Сколько есть времени у человека?
Аноним 12/05/16 Чтв 23:54:17 #280 №294005 
14630864573640.png
>>293998
Нихуя не понял, уловить момент не могу, что-то там мелькает, а на симуляции вроде нет.
Аноним 12/05/16 Чтв 23:57:44 #281 №294006 
>>294004
>Сколько есть времени у человека?
Секунд 15-20 будешь в сознании, затем отключишься из-за недостатка кислорода. Дальше в зависимости от того, как быстро тебя затащат в шлюз и окажут первую помощь, если за пару-тройку минут управятся, то без серьезных последствий для здоровья.
Аноним 13/05/16 Птн 00:01:09 #282 №294007 
>>294004
лицо начнет кипеть
Аноним 13/05/16 Птн 00:01:18 #283 №294008 
Посоветуйте что-нибудь интересное научно-популярное почитать про космос, чд етц.
Википедию уже раза 3 перечитал.
Аноним 13/05/16 Птн 00:02:07 #284 №294009 
Как на МКС выбрасывают мусор?
Аноним 13/05/16 Птн 00:07:14 #285 №294011 
>>294009
Запихивают в грузовики и сталкивают с орбиты. Все сгорает при входе в атмосферу и оседает тонким слоем у нас на головах.
Аноним 13/05/16 Птн 00:16:46 #286 №294012 
>>294008
Хокинг про все подряд, черные дыры, хуиры и прочее. Что именно - зависит от того, насколько ты в теме.
Попов - Суперобъекты, в основном про нейтронные звезды
Брайн Грин про суперструны, если очень захочется влезать в это. Но перед этим придется обмазаться Шинтаном Яу, иначе охуеешь. Да и от Яу ты охуеешь, че уж там

А, ну вот тут еще поройся, посмотри
http://www.astronomy.ru/forum/index.php/topic,48631.0.html
Аноним 13/05/16 Птн 01:13:22 #287 №294015 
>>294011
А отходы?
Аноним 13/05/16 Птн 01:14:34 #288 №294016 
Расскажите о квазизвёздных объедках.
Аноним 13/05/16 Птн 01:17:04 #289 №294017 
>>293994
>>293998
>В конце гифки звезда пролетает с ебанутой скоростью
Думаю, это просто артефакт конверсии симуляции в гифку. Это же не сырые данные с КЕКа, а нарисованная скриптом хуитка для иллюстрации. Что подтверждается тем, что на второй гифке все нормально кроме гиганской ЧД в центре

Аноним 13/05/16 Птн 01:41:18 #290 №294021 
>>294015
Какие именно? Из душа и туалета - перерабатываются, если ты об этом. Полученную воду используют в технических целях. Например для регенерации кислорода. Сухой "остаток" пихают в грузовики и отправляют нам на голову.
>>294016
Трудно что-то добавить к тому, что написано в википузии. Тебе механизм непонятен, или что?
Аноним 13/05/16 Птн 04:17:09 #291 №294026 
14631022297360.jpg
>>294021
Да именно механизм.
Вроде как говорилось, что маячит одиночный акреционный диск в который падают звёзды постоянно. Но что-то сомневаюсь. Особенно на фоне новости для меня о ближайшем квазаре который состоит из 2х СМЧД.
Аноним 13/05/16 Птн 08:42:09 #292 №294033 
>>294021
>Сухой "остаток" пихают в грузовики и отправляют нам на голову
То есть, можно сказать, что космонавты срут нам на головы.
Аноним 13/05/16 Птн 09:43:27 #293 №294035 
>>294033
Да. Но птицы делают это в миллиарды раз чаще, так что можно космонавтам и простить.
Аноним 13/05/16 Птн 10:23:04 #294 №294036 
>>294033
Да, в интервью как раз один из наших космонавтов шутил про это, но за давностью не помню кто именно.
Аноним 13/05/16 Птн 12:36:22 #295 №294043 
>>293938
https://www.youtube.com/watch?v=eQG9uZin3fA
Аноним 13/05/16 Птн 13:26:37 #296 №294045 
>>294026
Во-первых ты немного недопонял. Квазизвезда и квазар не одно и тоже. Квазизвезды, если таковые и были, существовали только на заре Вселенной, когда водорода было до сраки, а других элементов дальше лития не было совсем. По некоторым предположениям в ту эпоху могли существовать звезды совершенно пиздецового размера, такие, что даже знаменитая VY Canis Majoris была бы на фоне просто карланом - недоразумением. Внутри такой звезды, массой в несколько тысяч масс Солнца происходил коллапс ядра из-за слишком большой массы. При этом выделялась такое количество энергии, что внутренние слои врезались в вышерасположенные, но из-за колоссального давления верхних слоев такого взрыва и разлета материи, как при взрыве сверхновой не осуществлялось. При этом часть внутреннего вещества, начинала падать на ЧД и излучать как в аккреационном диске, при чем с такой силой, что давление внутреннего излучения удерживало верхний слой от обрушения в результате сжатия под силой тяжести. Наступало кратковременное, до миллиона лет равновесие. Получалась такая матрешка, сверху давил водород, а изнутри излучение от падающего вещества. Таким образом могли появляться черные дыры промежуточной массы.
Квазары - это ядра активных галактик, внутри которых уже присутствуют сверхмассивная черная дыра, или несколько таких. Возможно несколько черных дыр промежуточной массы сливались и образовывали первые сверхмассивные дыры, но может и нет. В любом случае - это не одно и тоже.
Аноним 13/05/16 Птн 18:07:54 #297 №294060 
>>294004
Главное выдохнуть, а то разорвет легкие или типа того. Кровь начнет кипеть. Ещё зависит от того, где ты вылетишь из шлюза. На земной орбите под лучами солнца/в тени Земли или далеко, там где температура ниже -200. Пару минут продержишься.
Аноним 13/05/16 Птн 19:15:34 #298 №294061 
>>294045
Не знал о таких. Спасибо.
Но я так понял АЯГ это не одно и тоже что и квазары. Значит ли что есть другие механизмы помимо ярой акреции или здесь только угол наклона влияет?
Будет ли 2 СМЧД пыхтеть сильнее?
Есть ли какие книжки, видео или статьи рассказывающих о них шире сухой википепии?
Аноним 13/05/16 Птн 19:34:07 #299 №294062 
14631572472710.jpg
Поясните залетному. В весь космос состоит из один и тех же элементов железл/золото/платина и пр.? Во всем космосе одни и те же законы физики ну то есть существует гравитация/давление и пр.?
Аноним 13/05/16 Птн 20:05:31 #300 №294064 
>>294045
Адекват в треде, я спокоен.
Аноним 13/05/16 Птн 20:14:02 #301 №294065 
>>294062
НА все вопросы ответ "да".
Аноним 13/05/16 Птн 20:26:36 #302 №294067 
>>294064
>Адекват
Т.е. чувак, высказавший очень узкоспечифичную гипотезу (еще и ошибся в ней раза 3)?
Аноним 13/05/16 Птн 20:27:40 #303 №294068 
>>294067
Прежде чем кукарекать, стоит написать что-то по существу.
Аноним 13/05/16 Птн 20:30:08 #304 №294069 
>>294068
Хуяссе, стоит? Точно, стоит? Ну вот ты и напиши.
Аноним 13/05/16 Птн 20:39:45 #305 №294070 
>>294061
>Но я так понял АЯГ это не одно и тоже что и квазары.
Я бы сказал, что квазары - это вид АЯГ.
Когда-то давно их нашли и долго не могли определить их природу, пока не появилась теория, выступающая за то, что квазары - это АЯГ. Я бы сказал, что в первую очередь квазары отличаются тем, что находятся очень далеко, у них огромное красное смещение. Самый близкий около 2-х миллиардов св. лет.
Само название означает QUASi-stellAR radio sources, то есть похожие на звезды радиоисточники, сейчас найдено огромное количество квазаров, которые не являются активными в радиодиапазоне. Так что со временем первоначальное определение и классификация менялась. Коротко можно сказать, что квазары - это вид наиболее удаленных и ярких АЯГ.
Аноним 13/05/16 Птн 22:02:48 #306 №294083 
14631661684510.png
Открыв гугл скай я не обнаружил полярные области.
Где норм фоточки галактик посмотреть?
Аноним 13/05/16 Птн 22:15:53 #307 №294086 
>>294070
А разве не мрк 231?
Аноним 13/05/16 Птн 22:45:38 #308 №294091 
>>294070
Как определили активные они в радиодиапазоне или нет?
Аноним 13/05/16 Птн 23:09:15 #309 №294098 
>>294083
https://2ch.hk/spc/res/276334.html
Аноним 14/05/16 Суб 00:03:04 #310 №294106 
>>294086
Да, Маркарян 231 ближайший. около 600 миллионов св. лет. Миль пардон.
Аноним 14/05/16 Суб 00:04:53 #311 №294108 
>>294091
Наводишь радиотелескоп на квазар и принимаешь радиоволны, или не принимаешь.
Аноним 14/05/16 Суб 01:27:12 #312 №294131 
>>294083
>Где норм фоточки галактик посмотреть?
https://www.spacetelescope.org/
http://hubblesite.org/
http://www.worldwidetelescope.org/
Аноним 14/05/16 Суб 03:40:08 #313 №294136 
14631864083270.gif
>>294131
>>294098
От души душевно в душу.
Аноним 14/05/16 Суб 05:00:18 #314 №294138 
Что представляет из себя Суперпустота Эридана?
Аноним 14/05/16 Суб 05:03:24 #315 №294139 
>>294138
>Что представляет из себя Суперпустота Эридана?
Суперпустоту. Смотреть в направлении на созвездие Эридана.
Аноним 14/05/16 Суб 06:03:14 #316 №294140 
>>294139
>Смотреть в направлении на созвездие Эридана
Тонко
Аноним 14/05/16 Суб 07:22:28 #317 №294141 
>>294140
>Тонко
Не, там не очень тонко получится. Трубень треба аграмадную, и стеклопакетов понпхать в енту трубень, и зыркнуть так. Короче, в добрый путь тебе, хлопец, в телескопотред, ежли узреть желаешь. Хотя шо там узревать, ежли там нихрена нет?
Аноним 14/05/16 Суб 10:19:48 #318 №294145 
Хочу обмазаться книгами по астрономии.
Реквестирую, соответственно, книги по сабжу, прочтение которых не станет пустой тратой времени.
Уточнение: совсем зеленый в астрономии.
Спасибо!
Аноним 14/05/16 Суб 15:02:19 #319 №294167 
>>294145
>Хочу обмазаться книгами по астрономии.
>Реквестирую, соответственно, книги по сабжу, прочтение которых не станет пустой тратой времени.

Учебник по астрономии за авторством Кононович и Мороз.
Аноним 14/05/16 Суб 15:21:24 #320 №294169 
>>294167
>Спасибо!

Аноним 14/05/16 Суб 17:02:59 #321 №294176 
Если солнце покраснеет, то какого цвета оно будет на закате? Увидит ли его глаз человека?
Аноним 14/05/16 Суб 17:17:46 #322 №294178 
>>294176
Красным и будет, каким же ещё?
Аноним 14/05/16 Суб 17:48:40 #323 №294179 
Какая существует самая точная карта солнечной системы?
Аноним 14/05/16 Суб 18:07:04 #324 №294181 
>>294179
Карты в обычном понимании нет, поскольку все объекты непрерывно движутся с переменными скоростями. Есть измеренные с высокой точностью результаты измерений орбитальных параметров для каждого тела, исходя из которых, а также с учетом взаимных возмущения, рассчитываются положения всех объектов на любой заданный момент времени.
Аноним 14/05/16 Суб 18:48:38 #325 №294183 
>>294181
А можно ссылки на каталоги этих параметров?
Аноним 14/05/16 Суб 19:30:49 #326 №294192 
>>294183
В википедию.
Аноним 15/05/16 Вск 00:53:51 #327 №294214 
Спейсаны, сегодня тян спросила, что за яркий объект рядом с луной был на небе в районе 21-00 подмосковье. Пришлось соврать, что венера, но ведь это не так. В общем, мне стыдно и как научится ориентироваться на ночном небе немножко хотя бы. Спасибо.
Аноним 15/05/16 Вск 00:55:06 #328 №294215 
>>294214
Научиться

Хотфикс
Аноним 15/05/16 Вск 00:58:04 #329 №294216 
>>294214
А это был юпитер
Аноним 15/05/16 Вск 01:22:18 #330 №294217 
>>294214
В принципе можно запомнить яркость планет. Звёзды не такие яркие. Венера только по утрам и вечерам, рядом с солнцем, редко, не везде. Почти всегда непонятная точка это Юпитер, но бывает и Сатурн. Марс ярко виден только в противостояния. Но в основном можно посмотреть в звёздной карте онлаен. В пределах месяца не сильно меняется.
Аноним 15/05/16 Вск 02:49:35 #331 №294220 
>>294178
Так же или по другому?
Аноним 15/05/16 Вск 08:21:59 #332 №294221 
>>294220
Закат будет ещё более красным.
Аноним 15/05/16 Вск 11:01:34 #333 №294227 
А если я куплю десять стократных телескопов и соединю их последовательно, у меня получится телескоп с увеличением триллиард раз? Поясните за подводные камни.
Аноним 15/05/16 Вск 11:24:58 #334 №294229 
>>294227
Если будет смотреть в телескоп с таким увеличением то будет очень большая вероятность что наткнешься на какую-нибудь очень далекую звезду в очень далекой галактике которая тебе выжгет глаз.
Аноним 15/05/16 Вск 13:48:44 #335 №294238 
14633093247380.jpg
>>292138 (OP)
> анонимные ученые мирового уровня
Каждый раз проигрываю.

А теперь, внимание, вопросы: как образуются звёзды второго и последующих поколений? Астрономы говорят, что из вещества уже погаснувших звёзд.
Но вот, например, сбросила звезда первого поколения немного вещества после превращения в сверхновую. Его ведь там действительно немного. Что с ним происходит дальше? Как из этого рассеянного облака образуется новая звезда? Вещества там явно недостаточно - почти вся масса осталась в белом карлике/нейтронной звезде/чёрной дыре. Но даже если достаточно, то притяжение звезды, сбросившей это вещество, не даст облаку покинуть пределы гравитационного поля этой звезды. А значит рядом с вновь образованной звездой должна находиться её "маман" (белый карлик, нейтройнная звезда, чёрная дыра). Рядом с Солнцем ничего такого нет.
Или вот ещё вопрос: когда звезда превращается в сверхновую ведь "выгорают" все элементы - которые могут "гореть" - вплоть до железа? Значит это вещество, что звезда сбрасывает, состоит целиком из железа? А если так, то как вообще из этого вещества могут образовываться нормальные звёзды?
Распишите процесс образования звёзд непервого поколения, пожалуйста. А то в гугле я нашёл только общую информацию, а ответов на вопросы возникшие в процессе чтения этой информации не нашёл.
Аноним 15/05/16 Вск 14:18:24 #336 №294240 
>>294227
Суть не только в увеличении но и в количестве собираемого света. Сколько телескопов последовательно не соединяй - света они будут собирать всего как один.
Аноним 15/05/16 Вск 16:03:30 #337 №294253 
>>294238
>Вещества там явно недостаточно - почти вся масса осталась в белом карлике/нейтронной звезде/чёрной дыре.
С ума сошёл? Большая часть вещества всегда уносится. На выродившееся ядро приходится в лучшем случае пятая часть массы звезды.
>Но даже если достаточно, то притяжение звезды, сбросившей это вещество, не даст облаку покинуть пределы гравитационного поля этой звезды.
Массой не вышла, не давать. Даже вещество сброшенной оболочки имеет скорость относительно центрального светила выше третьей космической для нормальной звезды, не то что для её огрызка.
>когда звезда превращается в сверхновую ведь "выгорают" все элементы - которые могут "гореть" - вплоть до железа?
Нет, конечно. Причина сверхновой коллапс ядра, а не коллапс звезды. Лёгких элементов во внешней оболочке более, чем достаточно.
Аноним 15/05/16 Вск 16:27:13 #338 №294256 
>>294240
А если тогда поставить много телескопов параллельно и подключить к одной камере?
Аноним 15/05/16 Вск 16:36:46 #339 №294260 
14633194063470.png
>>294253
Спасибо за ответ.
>Большая часть вещества всегда уносится. На выродившееся ядро приходится в лучшем случае пятая часть массы звезды.
Вроде бы нашёл подтверждение этим словам(pic), но как-то этого мало. Где почитать кроме википедии про сверхновые и рождение звёзд непервого поколения?

Да, с этим понятно. Спасибо ещё раз.
>Даже вещество сброшенной оболочки имеет скорость относительно центрального светила выше третьей космической
>Причина сверхновой коллапс ядра, а не коллапс звезды. Лёгких элементов во внешней оболочке более, чем достаточно.
Аноним 15/05/16 Вск 17:33:30 #340 №294269 
Вот смотрю я на отвеченный вопрос выше и таки решаюсть задать вопрос, что давно вертелся на языке.
Мои познания о звёздной эволюции на 90% подчерпнуты из книги Шкловского "Звёзды: их рождение, жизнь и смерть". Кто читал, поясните - насколько устарели данные там выкладки? Или с тем же успехом можно спросить, насколько устарели законы Ньютона?
Аноним 15/05/16 Вск 17:53:03 #341 №294272 
>>294256
Можно, но создание оптического интерферометра такая ебанистически сложная задача, что даже профессиональных таких телескопов полторы штуки. Короче забей.
Аноним 15/05/16 Вск 19:04:02 #342 №294278 
Почему нельзя сжать воду? Было бы забавно иметь 3 см брелок со сжатой там 5л воды.
Аноним 15/05/16 Вск 19:12:38 #343 №294279 
>>294278
Потому же почему нельзя сжать кусок твердого вещества. Мало места между молекулами.
Аноним 15/05/16 Вск 19:22:59 #344 №294280 
>>294278
Можно. Но она перестанет быть водой.
Аноним 15/05/16 Вск 19:37:27 #345 №294281 
>>294221
Т.е. при достаточной красности свет уйдёт в инфракрасный и его если и увидим то тусклее чем луна?
Аноним 15/05/16 Вск 22:34:33 #346 №294293 
14633408730610.gif
>>294278
Тебе придется заказывать для этого брелок с адамантиевыми стенками, способными выдерживать давление в несколько миллионов атмосфер. И да, это уже будет не вода, а высокотемпературный Лед-X.
Аноним 15/05/16 Вск 23:10:38 #347 №294296 
>>294293
>высокотемпературный Лед-X.
Уверен, что температура повысится? По уравнению Клайперона-Менделеева можно оценить, что температура уменьшится (вроде, можешь напихать хуёв и пояснить за это).
Аноним 15/05/16 Вск 23:18:25 #348 №294297 
>>294272
Ну, например есть Кек-1 и Кек-2 на Гавайях (Обсерватория Кека) (нет, это не даунская шутка). Те же бинокли - это оптические интерферометры, поэтому если взять два телескопа и аккуратно их соединить, то получим оптический интерферометр. Но больше увеличивать такой телескоп не станет, станет больше поле зрения и количество собираемого света в центре.
Аноним 15/05/16 Вск 23:21:40 #349 №294298 
>>294293
Есть предположения, где у нас в системе можеть быть такая среда?
Аноним 15/05/16 Вск 23:22:51 #350 №294299 
>>294298
Какая среда?
Аноним 15/05/16 Вск 23:25:17 #351 №294300 
>>294299
Лед Х
Аноним 16/05/16 Пнд 00:00:27 #352 №294303 
14633460280050.jpg
>>294300
В недрах Нептуна Урана. Однако с примесями .
Аноним 16/05/16 Пнд 00:01:00 #353 №294304 
14633460607880.jpg
>>294296
>Лед
>По уравнению Клайперона-Менделеева можно оценить
>можешь напихать хуёв
Менделеев с Клайпероном сами бы напихали тебе хуев полную панаму, если бы увидели, как ты используешь уравнение состояния идеального газа для получения выводов о жидкостях и кристаллах. И да, сжатие следует рассматривать как адиабатический процесс, работа которого тратится на нагрев рабочего тела.
>>294298
>где у нас в системе можеть быть такая среда
Где-то в глубинах ледяной мантии пикрелейтеда.
Аноним 16/05/16 Пнд 00:03:33 #354 №294307 
>>294303
Дорогой наверное пиздец. Вот бы его там накопать и привезти сюда
Аноним 16/05/16 Пнд 00:05:37 #355 №294309 
>>294307
Давление в миллионы атмосфер по дороге на Землю в анусе будешь поддерживать? Купи себе просто алмазную наковальню, и будет тебе счастье.
Аноним 16/05/16 Пнд 00:29:55 #356 №294318 
>>294269
>книги Шкловского "Звёзды: их рождение, жизнь и смерть"
>насколько устарели данные там выкладки?
В целом, практически все там описанное сохраняет актуальность. Из явно устаревшего могу назвать только главу о причинах аномально низкого потока солнечных нейтрино, где перечислены всякие манягипотезы, поскольку на то время нейтринные осцилляции, за которые в прошлом году дали Нобелевку, еще не были обнаружены. Также совсем ничего нет по коричневым карликам, что тоже понятно — первый из них выявили только в 1995. В остальном же это, можно сказать, почти классика для своего уровня.
Аноним 16/05/16 Пнд 00:31:27 #357 №294319 
>>294309
Вот это сейчас непонятно было
Аноним 16/05/16 Пнд 00:33:37 #358 №294321 
>>294319
Лед-X может существовать только при выских давлениях, если давление упадет до атмосферного, он разуплотнится до обычного льда-I.
Аноним 16/05/16 Пнд 09:11:24 #359 №294338 
Объясните доступным языком, что такое приливные силы и какое у них действие.
Аноним 16/05/16 Пнд 09:27:44 #360 №294339 
>>293937
Ну платину уже пора бы совать в пастебиновский фак, про воздушные старты, в космос без скафандра, черные дыры, зачем МКС, почему не летаем на Луну и так далее.
Аноним 16/05/16 Пнд 09:59:21 #361 №294340 
>>294321
В чем проблема забрать его вместе с давлением?
Аноним 16/05/16 Пнд 09:59:59 #362 №294341 
14633819994590.png
>>294338
Силы возникающие в следствии нарушения однородности гравитационного поля одного небесного тела другим. У планеты-океана без спутника вся поверхность воды будет отстоять от центра на одинаковом расстоянии. Стоит появится спутнику, как планета начнет вращаться не только вокруг своей оси, но и общего центра масс в системе спутник-планета. Это значит, что отцентровые силы на воду с противоположной от спутника стороны увеличатся, в следствии сложения скоростей. Само существование спутника уменьшит силу тяжести на стороне планеты близкой к спутнику. Возникнет перераспределение воды, как на пике (википедия©).

https://ru.wikipedia.org/wiki/%D0%9F%D1%80%D0%B8%D0%BB%D0%B8%D0%B2%D0%BD%D1%8B%D0%B5_%D1%81%D0%B8%D0%BB%D1%8B
https://ru.wikipedia.org/wiki/%D0%9F%D1%80%D0%B8%D0%BB%D0%B8%D0%B2%D0%BD%D0%BE%D0%B5_%D1%83%D1%81%D0%BA%D0%BE%D1%80%D0%B5%D0%BD%D0%B8%D0%B5_%D0%9B%D1%83%D0%BD%D1%8B
Аноним 16/05/16 Пнд 10:02:00 #363 №294342 
>>294340
Способы поддержания такого давления в студию.
Аноним 16/05/16 Пнд 10:07:46 #364 №294344 
>>294342
Есть ли смысл брать этот Лед-Х, даже если мы сможем поддержать огромное давление? Очень плотный, нестабильный материал, но держащийся только с давлением, которое ещё поддерживать надо, а это дохера дорого
Аноним 16/05/16 Пнд 10:09:12 #365 №294345 
>>294338
На орбите 500км высотой гравитация такая-то.
На орбите 501км гравитация на пипирку меньше.
Соответственно у 1км палки, болтающейся на орбите, нижнюю часть будет тянуть к планете сильнее верхней, это и есть приливная сила. Если палка вращается, то рано или поздно приливная сила её остановит.
Почему приливная: если поместить на орбиту гондон с водой и раскрутить его, то эта сила будет его слегка плющить, создавая неравномерность формы. У Земляшки, которая представляет собой такой же водяной гондон, гравитация Луны точно таким же образом вызывает неравномерность глубины океанов, и получаются приливы и отливы.
Аноним 16/05/16 Пнд 10:11:34 #366 №294346 
>>294340
В том, что для такого давления нужна планета, гравитационно сжимающая это говно другими говнами, поэтому придётся забирать вместе с планетой.

Искусственно такое можно поддержать только между алмазными наковальнями в течение микросекунд в объемах комариного хуя.
Аноним 16/05/16 Пнд 10:16:34 #367 №294347 
>>294341
>>294345
Благодарю.
Аноним 16/05/16 Пнд 10:22:50 #368 №294348 
>>294342
Силовое поле какое-нибудь...
Аноним 16/05/16 Пнд 10:38:05 #369 №294350 
14633842852160.jpg
>>294348
Аноним 16/05/16 Пнд 12:42:42 #370 №294360 
>>294339
Блять, вот все промолчали, но ты то влез, да? Был сходняк спейсачеров несколько тредов назад, кворум имел место быть, порешили, что никто из залетных ни шапки читать не будет, ни тем более какие сторонние факи. Да и суть треда все же в ответах на любые вопросы, как бы не горело от круговорота черных дыр вокруг вышедшего голышом в открытый космос долбоеба. Так что завязывай.Никто не мешает запилить тебе фак самому, чтобы убедиться в его абсолютной невостребованности
Аноним 16/05/16 Пнд 13:06:52 #371 №294362 
>>294360
суть не в том, что они читать будут, можно самим не писать каждый раз ответ на их вопросы по новому, а просто копировать копипастный ответ.
Аноним 16/05/16 Пнд 13:12:10 #372 №294363 
>>294360
Этого поддерживаю в этом:
>никто из залетных ни шапки читать не будет, ни тем более какие сторонние факи.
>>294362
Однако, вот это тоже не лишено смысла:
>самим не писать каждый раз ответ на их вопросы по новому, а просто копировать копипастный ответ.
Все равно делать нехуй, хоть движ будет. Готов по мере сил поучаствовать.
Можно даже тред отдельный сделать. Платиновые вопросы и ответы, для обсуждения и причесывания ответов на платину, плюс на собирание ссылок, что бы как у взрослых.
Аноним 16/05/16 Пнд 13:52:47 #373 №294366 
>>294350
Ну такая еба с марса будет дохуя стоить. Можно кулончик сделать и продать миллиардеру
Аноним 16/05/16 Пнд 15:31:14 #374 №294372 
>>294366
Только какому ни будь Бонду. Малейший дисбаланс в давлении и этот ёба кулон взрывает здание.
> Можно кулончик сделать и продать квадриллионеру
подправил малёх.
Аноним 16/05/16 Пнд 16:58:07 #375 №294378 
>>294372
>Малейший дисбаланс в давлении и этот ёба кулон взрывает здание
Одним зданием дело не ограничится, потенциальная энергия такой хуитки будет доходить до килотонны в тротиловом эквиваленте.
Аноним 16/05/16 Пнд 18:03:43 #376 №294382 
>>294378
https://www.youtube.com/watch?v=n0KEY9p3jZI
Смотря какое здание.
Аноним 16/05/16 Пнд 20:11:24 #377 №294394 
>>294382
Деревеньку разрушит на изи, но бункер может выдержать
Аноним 17/05/16 Втр 00:53:14 #378 №294428 
>>294318
Спасибо, анон. Про то, что бурые карлики из себя представляют я, в принципе, на момент прочтения уже знал. А вот про нейтрино надо будет ещё поискать-почитать
Аноним 17/05/16 Втр 02:07:53 #379 №294459 
14634400731730.png
Как сделать так что бы кусок скалы с поверхности планеты улетел в войд без порталов.ь?
Аноним 17/05/16 Втр 03:16:51 #380 №294462 
>>294145
Элементарная астрономия. Климишин
Что и как наблюдать на небе. Цесевич
Вращение Земли. Куликов

Если вообще 0 в астрономии, то для вхождения Новая занимательная астрономия Комарова. Самое то - небольшие статьи-рассказы на 2-3 страницы про всё, что связано с космосом.
Аноним 17/05/16 Втр 07:51:52 #381 №294464 
>>294462
Да пусть в спейсаче посидит месяцок. Будет квантовать гравитоны в уме.
Аноним 17/05/16 Втр 20:53:31 #382 №294528 
Астрономы, вопрос.
Что стоит почитать, откуда начинать и чему учиться, что бы суметь с помощью говна, палок и телескопа рассчитывать хоть что нибудь? Массу, период обращение, полуоси, эксцентриситеты, размер, координаты - что угодно, лишь бы получить самому циферку. И я не технарь по образованиюпсихологи не гуманитарии
Аноним 17/05/16 Втр 20:57:46 #383 №294530 
>>294528
Начни с трудов Гиппарха и Пифагора.
Аноним 17/05/16 Втр 23:42:13 #384 №294543 
>>294528
Тебе же это нахуй не надо.
Аноним 17/05/16 Втр 23:56:41 #385 №294547 
14635186010770.png
>>294528
Я хуею. Пе­ре­льм­ан в спам-листе.
Аноним 18/05/16 Срд 00:11:18 #386 №294551 
Путь фотона конечен? Если я посвечу фонариком в космос значит ли это я увековечил себя в простанстве-времени на миллиарды лет?
Аноним 18/05/16 Срд 00:12:21 #387 №294552 
>>294547
А нехер было в Швецию уезжать!
Аноним 18/05/16 Срд 00:51:23 #388 №294560 
>>294551
>увековечил себя
>посветив фонариком
Ты скорее увековечишь себя, отразив свет от своей тушки.
Аноним 18/05/16 Срд 01:41:40 #389 №294569 
Посоны!
Для курсача нужно узнать количество живых спутников дохуя вытянутых орбитах с большим наклонением (хочется обосновать тезис о том, что большинство из них летают или на орбитах, достаточно близких к экваториальной, или на полярной, всяких изъёбистых вроде "Молнии" мало). Идей насчет того, где искать такую информацию - 0. Есть всякие красивые карты, которые наглядно показывают, что это так, но таких циферок/фильтров по наклонению там нет. А нужно.
Аноним 18/05/16 Срд 01:42:17 #390 №294570 
>>294569
>на дохуя вытянутых орбитах
квикфикс
Аноним 18/05/16 Срд 02:21:19 #391 №294571 
>>294569
>живых спутников
>вытянутых орбитах
>большинство из них летают
Ну хуй знает.

>но таких циферок/фильтров по наклонению там нет. А нужно.
Если очень нужно, то берешь свои данные из одного очень проверенного источника - от себя. Как будто препод будет проверять все источники, тем более если это будут ссылки. Главное не переборщить. Это классика курсачей, это знать надо.
Аноним 18/05/16 Срд 02:39:45 #392 №294572 
>>294571
>Ну хуй знает.
Если посмотреть на карты, выдет так, что искомых сильно вытянутых орбит с наклонением под 45 очень мало.
>Это классика курсачей
Но я-то курсач пишу для себя в первую очередь, для души. Хочу всё логично и точно разложить, а не картиночками.

В общем, если кто знает/видел инфу - с меня нихуя.
Аноним 18/05/16 Срд 02:43:22 #393 №294573 
>>294572
http://www.ucsusa.org/nuclear-weapons/space-weapons/satellite-database

>a listing of the more than 1000 operational satellites currently in orbit around Earth
Аноним 18/05/16 Срд 03:55:00 #394 №294574 
>>294573
О. Спасибо. Еще и в удобном для сортировки формате. Цистерны ксенона тебе.
Аноним 18/05/16 Срд 08:09:10 #395 №294576 
Говорят, что ложементы в "Союзах" анатомические до безобразия. Значит ли это, что они изготавливаются и под запасной экипаж, и в случае чего меняются прямо на старте? Или дублеров подбирают с очень похожими параметрами тела?
Аноним 18/05/16 Срд 08:39:37 #396 №294578 
>>291931 (OP)
1
Аноним 18/05/16 Срд 08:52:40 #397 №294580 
>>294576
Индивидуально для каждого члена экипажа.
Ищи видео, как их гипсом заливают при изготовлении ложементов.
Аноним 18/05/16 Срд 14:37:08 #398 №294609 
Если я окажусь в центре сверхвойда (Реликтовое холодное пятно), то без фонарика я не увижу даже собственных рук? Там абсолютная темнота или человек может увидеть далёкие звёзды? Что вообще со мной будет?
Протяжённость сверхвойда - 1 млрд. световых лет. Просто 1 млрд. световых лет пустоты.
Аноним 18/05/16 Срд 15:29:12 #399 №294616 
>>294609
Глазом нихуя не увидишь. Андромеду то едва видно невооруженным глазом, а она всего в 2.5 миллионах св. лет. Глаз - хуеватый прибор для разглядывания таких далёких объектов.
Аноним 18/05/16 Срд 15:46:17 #400 №294617 
>>294609
Ничего, там галактики даже есть.
Аноним 18/05/16 Срд 16:09:49 #401 №294618 
Где можно посмотреть стримы онлайн с космических телескопов?
Аноним 18/05/16 Срд 16:11:58 #402 №294619 
>>294618
Нигде, они не транслируют видео, а снимают фото.
Аноним 18/05/16 Срд 16:15:25 #403 №294620 
>>294619
>>294618
> стримы онлайн с космических телескопов
Я прямо так и забил в гугель и о чудо!

http://original.livestream.com/astroeduca
наебалово?

Насколько я понимаю, трансляция не 24/7, так что помониторь денёк-другой. а вообще подозрительно трансляция выглядит
Аноним 18/05/16 Срд 16:19:08 #404 №294621 
>>294620
А с чего ты взял, что это трансляция с космического телескопа? Там например на сайте вот такая строчка есть.
>Observations from Centro Astronomico Roque Saucillo (Vega de San Mateo - Gran Canaria)
Аноним 18/05/16 Срд 16:22:52 #405 №294622 
14635777721170.png
>>294621
мог бы просто представить, будто это с хабла транслируют
Аноним 18/05/16 Срд 16:45:58 #406 №294627 
>>294609
> Если я окажусь в центре сверхвойда (Реликтовое холодное пятно), то без фонарика я не увижу даже собственных рук? Там абсолютная темнота или человек может увидеть далёкие звёзды? Что вообще со мной будет?
Во-первых, пятно это пятно, а войд это войд. Не путай. Во-вторых, там просто плотность галактик пониже. да и всё. Находясь в галактике, будет как у нас, вне галактики - чуть менее "яркое" небо. Учитывая, что вне галактики и у нас темно как у негра в жопе, разницы - никакой.
Аноним 18/05/16 Срд 16:48:22 #407 №294628 
>>294619
Смотря какие. Некоторые стакают кадры прямо с видео.

>>294618
Нигде, но онлайн можно заказать время на телескопах, есть несколько сервисов.
Аноним 18/05/16 Срд 17:24:11 #408 №294632 
>>294628
>Смотря какие. Некоторые стакают кадры прямо с видео.

И какие же орбитальные телескопы снимают видео, фантазер ебаный?
Аноним 18/05/16 Срд 17:48:28 #409 №294638 
>>294632
А, космических. Это я проворонил. Так-то и об онлайне никакой речи быть не может, очередь такая что научно-технический совет требуется, принимает решение о приоритетах.

но вообще прежде чем с апломбом что-то заявлять, тебе стоило бы разобраться что такое временная задержка и накопление в матрицах аппаратов для съемки поверхности, например. Это не стекинг видеокадров и не дипскай, но принцип увеличения snr принципиально тот же, хоть и аппаратно реализован иначе
Аноним 18/05/16 Срд 18:15:30 #410 №294640 
Какая там вторая космическая для Млечного пути? В вики нет ответа.
Аноним 18/05/16 Срд 18:23:59 #411 №294643 
>>294640
https://ru.wikipedia.org/wiki/%D0%A7%D0%B5%D1%82%D0%B2%D1%91%D1%80%D1%82%D0%B0%D1%8F_%D0%BA%D0%BE%D1%81%D0%BC%D0%B8%D1%87%D0%B5%D1%81%D0%BA%D0%B0%D1%8F_%D1%81%D0%BA%D0%BE%D1%80%D0%BE%D1%81%D1%82%D1%8C
Аноним 18/05/16 Срд 18:50:17 #412 №294648 
Когда Солнце начнет раздуваться, то орбиты планет никак не изменятся и раздувающаяся атмосфера будет их поглощать, или же они будут отдаляться?
Аноним 18/05/16 Срд 18:54:53 #413 №294649 
>>294648
Масса солнца не будет изменяться -> планеты останутся на месте.
Аноним 18/05/16 Срд 18:59:57 #414 №294650 
>>294648
Будет изменяться размер оболочки, плюс часть оболочки будет активно теряться -> планеты будут отдаляться от солнца.
Аноним 18/05/16 Срд 19:01:27 #415 №294651 
>>294649
>Масса солнца не будет изменяться -> планеты останутся на месте.
Только если звезду можно рассматривать как материальную точку. Сильно сомневаюсь, что такое катит.
Аноним 18/05/16 Срд 19:04:58 #416 №294654 
>>294651
Центр масс все равно там же окажется.
Аноним 18/05/16 Срд 19:05:40 #417 №294655 
>>294654
Центр масс сместится.
Аноним 18/05/16 Срд 19:21:49 #418 №294658 
>>294655
Куда?
Аноним 18/05/16 Срд 19:30:17 #419 №294663 
>>294658
Не куда, а откуда.
От центра Солнца.
Аноним 18/05/16 Срд 19:31:41 #420 №294664 
>>294663
Можешь нарисовать? Так-то это, возможно, тянет на Нобелевку.
Аноним 18/05/16 Срд 19:32:46 #421 №294665 
>>294664
Это не тянет на Нобелевку, это блядь очевидная хуета.
Аноним 18/05/16 Срд 19:34:23 #422 №294667 
>>294569
Ты не умеешь в гугол или в ингриш? Первая же ссылка http://www.ucsusa.org/nuclear-weapons/space-weapons/satellite-database
Аноним 18/05/16 Срд 19:34:24 #423 №294668 
>>294665
Потому и говорю: нарисуй. Куда может уехать центр масс сферически симметричного тела.
Аноним 18/05/16 Срд 19:40:32 #424 №294672 
А если пирожок побудет с пару минут в открытом космосе, его можно будет потом есть на Земле?
Аноним 18/05/16 Срд 19:42:04 #425 №294673 
>>294672
Ты и собаку из космоса...
Аноним 18/05/16 Срд 19:44:59 #426 №294674 
>>294673
Так можно? Облучение там, не вредно будет?
Аноним 18/05/16 Срд 19:51:11 #427 №294678 
>>294668
>Потому и говорю: нарисуй
Извини, бисер закончился.
Если Солнце потеряет часть массы, (а оно потеряет) то блядь, посмотри в формулу всемирного тяготения и сделай выводы самостоятельно.

Второе: При раздувании в красный гигант солнечное вещество размажется на 100 млрд километров, а не будет компактно собрано, там, где оно сейчас. Рассматривать Солнце, как точечный объект у которого вся масса сосредоточена в этой самой центральной точке не стоит, чай не в восьмом классе.

Алсо:
http://www.sciencemag.org/news/2011/09/earths-ultimate-destruction%E2%80%94and-possible-reincarnation
Аноним 18/05/16 Срд 19:51:43 #428 №294679 
>>294674
Не, за две минуты даже в радиационных поясах нихуя не будет, а вот за две минуты на солнечной стороне может чутка перегреться(не на много) сразу кушать пожалуй не стоит.
Аноним 18/05/16 Срд 19:53:33 #429 №294680 
>>294672
>побудет с пару минут в открытом космосе его можно будет потом есть на Земле?
Можно, ешь на здоровье. Бабушке только передай, что разогревать пирожки в микроволновке дешевле и быстрее.
Аноним 18/05/16 Срд 19:56:12 #430 №294683 
>>294678
А сколько Солнце массы потеряет? Хватит, чтобы Юпи поднял массу и засиял коричневым карликом?
Аноним 18/05/16 Срд 19:59:38 #431 №294685 
>>294683
Нет не хватит, и вообще Юпитер там мало чего получит, если вообще получит, он ебать далеко.
Аноним 18/05/16 Срд 20:01:58 #432 №294687 
>>294674
Облучение в плане наведенной радиации опасно только если оно нейтронное. Нейтроны от Солнца дальше, чем на миллион километров не улетают догадайся, почему Поэтому не вредно.
Аноним 18/05/16 Срд 22:01:26 #433 №294707 
>>294678
Ок, признаю, обосрался, недооценив долю теряющейся массы.
> Рассматривать Солнце, как точечный объект у которого вся масса сосредоточена в этой самой центральной точке не стоит, чай не в восьмом классе.
Не стоит только для планет, которые окажутся внутри. ЦМ Солнце-Юпитер, например, сместится на совсем малую величину.
Аноним 18/05/16 Срд 22:08:26 #434 №294711 
>>294707
>Продолжает считать, что масса сосредоточена в одной точке.
С упоротыми смысла спорить не вижу. Считай как хочешь.
Аноним 18/05/16 Срд 22:28:56 #435 №294717 
>>294711
И какую же погрешность даст такое упрощение для Юпитера и Солнца-красного гиганта?
SOI Аноним 19/05/16 Чтв 00:34:02 #436 №294740 
14636072426950.png
Анон, поясни за сферу тяготения для двойных систем. При значительной массе спутника она каким-то хуем становится больше сферы хилла, и при почти равных массах радиус компаньона доходит ажно до центра более массивной. Как это кореллирует с
>Модель сопряжённых конических сечений применима лишь внутри сферы действия тяготения
? Как может быть орбита относительно спутника эллиптической, если в апоцентре корабль может подлететь почти к центру более массивной?
Аноним 19/05/16 Чтв 03:07:18 #437 №294745 
>>294678
>100 млрд километров
У тебя с три лишних разряда вышло.
Аноним 19/05/16 Чтв 03:10:07 #438 №294746 
14636166075570.jpg
>>294687
>догадайся, почему
Можно я попробую?!
Аноним 19/05/16 Чтв 03:33:51 #439 №294747 
Почему нет квазаров на малом красном смещении? Где об этом почитать?
Аноним 19/05/16 Чтв 08:05:43 #440 №294752 
>>294747
Недостаточно далеко. Почитай что такое квзар буквально.
Аноним 19/05/16 Чтв 08:08:54 #441 №294754 
>>294752
>Недостаточно далеко.
Я об этом и спрашиваю. Почему нет близких квазаров?
Аноним 19/05/16 Чтв 08:58:41 #442 №294757 
>>294740
Ну во-первых, тебя смутило название "сфера". Это условное название. Оно сложной формы, на самом деле. Это область, в которой одно тело тянет к себе объект сильнее, чем остальные.
>Как может быть орбита относительно спутника эллиптической, если в апоцентре корабль может подлететь почти к центру более массивной?
Никак. По кеплеровским эллипсам объекты летают только в учебниках и KSP. Вылетел из одной сферы, т.е. пересёк границу, в которой уравниваются силы гравитации разных тел - перешел в какую-нибудь другую, только и всего.
Аноним 19/05/16 Чтв 09:06:08 #443 №294758 
>>294754
Потому что на малом красном смещении это уже не квазар? По определению это ядро галактики, которое светит _достаточно_издалека_
Аноним 19/05/16 Чтв 09:13:34 #444 №294760 
>>294758
>Потому что на малом красном смещении это уже не квазар? По определению это ядро галактики, которое светит _достаточно_издалека_
Ок, перефразируем. Почему на малом ККС не наблюдается таких же объектов, как квазары?
Аноним 19/05/16 Чтв 12:19:25 #445 №294766 
>>294754
>Почему нет близких квазаров?
Потому что они были очень давно и сейчас стали обычными галактиками.
Аноним 19/05/16 Чтв 16:23:01 #446 №294789 
Почему отделяющиеся от ракеты ступени падают на Землю, а не продолжают двигаться вверх вместе с ракетой?
Аноним 19/05/16 Чтв 16:38:02 #447 №294791 
>>294789
Сила притяжение Земли жи есть.
Аноним 19/05/16 Чтв 16:56:14 #448 №294793 
А реально вообще сделать термоядерную бомбу мощностью в гигатонну или тератонну? На случай уничтожения опасного астероида? Или же для таких мощностей понадобится бомба на антиматерии?
Аноним 19/05/16 Чтв 17:19:13 #449 №294795 
>>294793
Испытания Царь бомбы как раз проводили, что бы подтвердить теорию, что нет ограничения на мощность водородной бомбы. Подтвердили чо
Аноним 19/05/16 Чтв 17:39:50 #450 №294799 
>>294793
Представь что будет, если ракета не взлетит и пизданется на Землю.
Аноним 19/05/16 Чтв 17:48:59 #451 №294803 
>>294766
>Потому что они были очень давно и сейчас стали обычными галактиками.
Т.е. ~2.5 млрд. лет назад появился последний такой, и больше не появлялось? 10 млрд. лет подряд появлялись, а потом закончились. Почему?
Аноним 19/05/16 Чтв 17:58:21 #452 №294807 
>>294803
Активное пожирание ядром звёзд жи. Звёзды кончились возле ядра, масса распределилась равномерно. В целом хуйзнает почему так, нужен закон распределения масс в галактике чтобы пояснить
Аноним 19/05/16 Чтв 18:10:00 #453 №294808 
>>294803
Водород закончился возле ядер. Перестал падать на ЧД в огромных количествах. Поэтому, кстати в балджах превалируют старые звезды, процессы звездообразования там практически на нуле.
Аноним 19/05/16 Чтв 18:29:49 #454 №294812 
>>294808
>Водород закончился возле ядер
Ну и что, ни у одной галактики больше с тех времен водород не кончался?
Аноним 19/05/16 Чтв 19:04:35 #455 №294822 
>>294799
Активную часть бомбуя изъять и положить в новую ракету. Это ж не бак с нитроглицерином, чтоб от падения бахать.
Аноним 19/05/16 Чтв 19:06:33 #456 №294823 
>>294812
Водород закончился и перестал фонить. А у тех вдали тоже закончился, но мы пока еще принимаем фон из далеких миллиардов лет прошлого.
Аноним 19/05/16 Чтв 19:09:27 #457 №294824 
>>294799
>>294822
>бак с нитроглицерином
Допустим, было бы что-то на подобии этого. Может ли это взорваться от дикой вибрации при взлёте ракеты?
Аноним 19/05/16 Чтв 19:33:21 #458 №294828 
>>294793
>На случай уничтожения опасного астероида?
Ну взорвал ты бомбу на поверхности астероида, что дальше? Денется-то он куда?
Аноним 19/05/16 Чтв 19:37:32 #459 №294830 
>>294828
Еба - гигатонна это не хуй собачий. Даже если на поверхности взорвется, то это как столкновение с астероидом такого же размера.
Аноним 19/05/16 Чтв 19:38:34 #460 №294832 
>>294828
Он имел ввиду, наверно, мол разбиение астероида на более мелкие и менее опасные куски
Аноним 19/05/16 Чтв 19:56:46 #461 №294835 
>>294824
Более чем, нитроглицерин в чистом виде очень нестабилен. Собственно доставить его каким либо образом до стартовой площадки для столь ебанутого эксперимента - уже нетривиальная инженерная задача.
Аноним 19/05/16 Чтв 20:25:14 #462 №294842 
>>294832
А смысл? Будет просто менее предсказуемый набор нихуя не мелких обломков, с которым уже точно ничего не сделаешь. Это вообще при условии что оно развалится, поражающие факторы ТЯО в вакууме не раскрываются совершенно, даже гигатонна на поверхности будет как пердёж. Только если шахту пробурить или с нескольких сторон подорвать, и то хз что получится.

Астероиды надо сводить с траектории, например покраской, за счёт эффекта Ярковского он изменит траекторию, правда куда - большой вопрос. А боньбы вообще не решат проблему, даже для мелкого астера.
Аноним 19/05/16 Чтв 20:31:18 #463 №294843 
>>294842
Там столько породы испарится с поверхности, что реактивный эффект от этого будет сравним с эффектом от покраски.
Аноним 19/05/16 Чтв 20:31:24 #464 №294844 
Спейсач, а как разгоняться гравитационным маневром? Имеет ли смысл, когда корабль сам способен развивать ускорение до 10g продолжительное время? Например, гравитационный маневр у Юпитера может дать дополнительно 42 км/с, но двигаясь на родных 10g, корабль за 10 суток сможет набрать скорость в 85 000 км/с. Да хоть на 1g, - 8 500 км/с, тоже намного больше, чем какие-то 42 км/с. Для такого кораблика есть смысл проводить какие-то там хитроумные маневры, или просто нецелесообразно?
Аноним 19/05/16 Чтв 20:36:12 #465 №294846 
>>294843
Сомневаюсь. Каменюка весит пиздец как дохуя. Вопрос в запасённой энергии, даже при полном её преобразовании в реактивное движение (чего не будет) миллиард тонн ТНТ это слишком мало для придания хоть сколько-нибудь заметной скорости хоть сколько-нибудь приличному астеру. И уж точно менее предсказуемо, на кого Кеплер пошлёт. Краска даст гораздо больше.
Аноним 19/05/16 Чтв 20:39:11 #466 №294849 
>>294844
Если у тебя бесконечное топливо, многое в современной космонавтике теряет смысл.
Аноним 19/05/16 Чтв 20:46:42 #467 №294854 
>>294846
>миллиард тонн ТНТ
Уже ТНТ? Там же до этого термоядерный боеприпас вротебенной мощности был изначально?
>Каменюка весит пиздец как дохуя.
>Краска даст гораздо больше.
Охуительно, испарение дохуя тысячтонн с поверхности ничего не даст, а краска даст. Иди ты нахуй, пожалуйста.
Аноним 19/05/16 Чтв 20:47:07 #468 №294855 
>>294844
>Спейсач, а как разгоняться гравитационным маневром?
Летишь в планету.
Пролетаешь перед ней - замедляешься.
Пролетаешь позади неё - ускоряешься.
Максимальный прирост/замедление составляет две орбитальные скорости планеты, в случае разворота вокруг планеты на 180 и подлёта по касательной.

>Имеет ли смысл, когда корабль сам способен развивать ускорение до 10g продолжительное время?
Ускорение не имеет значения (если манёвр не ограничен по времени). Вопрос бессмысленен. Разберись что такое характеристическая скорость.

>85 000 км/с
Если у тебя 85000км/с дельты и ускорение 10g, тебе вообще нихуя не нужно, даже орбит как таковых. Можешь тупо над Юпитером без всяких орбит висеть и сэмки вниз плевать, на щупальца юпитериан. А лучше лети на своём вакуумном вертолёте поколений осваивать /sf/.
Аноним 19/05/16 Чтв 20:56:42 #469 №294859 
>>294855
>вакуумном дирижабле
Аноним 19/05/16 Чтв 21:09:15 #470 №294862 
14636813561060.gif
>>294854
>Уже ТНТ?
Гигатонна, если что, это тротиловый эквивалент, т.е. столько тонн тротила надо взорвать, чтобы получить ту же энергию. Весьма условная цифра. И кстати от тротила там больше проку даже будет, т.к. он хоть рабочим телом послужит

>Охуительно, испарение дохуя тысячтонн с поверхности ничего не даст
Практически нихуя. Во-первых, каменюка диаметром например 300м может весить в районе 30 млн. тонн. (в зависимости от состава). Можешь сам посчитать сколько энергии тебе надо, чтобы сдвинуть её на банальный 1м/с, и немножечко охуеть, переведя свою сраную гигатонну в джоули и сравнив. Во-вторых, на реальное дело уйдет лишь мизерная часть от энергии этой гигатонны, т.к. половина уйдет в пустоту, а испарение породы будет крайне малозаметным - шарика-то нет, за отсутствием атмосферы или оболочки.

Но это все хуйня, самое главное что выделение энергии единовременное, и нет никакой гарантии что полученная траектория будет стабильной. А краска будет лежать веками и давить постоянно.
Аноним 19/05/16 Чтв 21:12:55 #471 №294863 
>>294862
Т.е как фильме с Брюсом Уиллисом, нужно бурить астероид и закладывать заряд в глубину?
Аноним 19/05/16 Чтв 21:19:03 #472 №294865 
>>294863
Ну да, от подземного взрыва с направленным выбросом породы и испарившихся говн, как из пушки, побольше толку будет. Но все равно мало, и все равно принципиальное ограничение единомоментности не преодолеть.
Аноним 19/05/16 Чтв 21:25:28 #473 №294866 
>>294862
При чем тут атмосфера вообще, ты отбитый совсем?
>сдвинуть её на банальный 1м/с,
Так много вообще не нужно.
>А краска будет лежать веками
Ага, а астероид такой: "ну ок, подожду век другой, не буду въебениваться."
Алсо, ты как красить собрался? Таджиков отправишь?
Аноним 19/05/16 Чтв 21:29:31 #474 №294868 
Погодите, в открытом космосе могут происходить взрывы? Но как? Если я правильно понимаю, в любом взрыве присутствует воспламенение -> воспламенению нужна "газовоздушная смесь" -> в открытом космосе нет кислорода.
Или есть какие-то кислородосодержащие супер вещества?
В общем, по хардкору объясните зеленому.
Аноним 19/05/16 Чтв 21:30:48 #475 №294869 
>>294862
Я сомневаюсь, что если кто-то будет сооружать гигатонную бомбу, то он почему-то не задумается о максимальном применении энергии от взрыва. Перед атакой скорее всего будет проведен общий анализ астероида и найдено оптимальное место для взрыва, какая-нибудь слабая точка. Но перед самой боеголовкой в точку пошлют баллистический заряд, который пробьет отверстие или углубление, в которое уже и войдет ракета-бомба.

А можно создать оболочку бомбы так, чтобы при взрыве мы получили что-то вроде плазменного копья, а не распространения во все стороны?
Аноним 19/05/16 Чтв 21:53:02 #476 №294873 
>>294866
>При чем тут атмосфера вообще
А за счет чего происходит испарение породы по-твоему? За счет только излучения? В случае поверхностного взрыва в вакууме большая его часть уйдет на всё что угодно, только не на испарение.

>Так много вообще не нужно.
Ну как сказать. Может быть для какого-то хитрого манёвра хватит и 1м/с, но вообще чтобы тупо поднять или опустить орбиту у околоземного и гарантированно не быть зависимым от эволюции траектории - надо реально дохуя. Малой дельтой можно обойтись только для сиюминутного избежания окон гарантированного столкновения, как правило они несколько десятков-сот метров шириной, но и определить что астероид точно направится в него, можно только незадолго до влёта, в силу неопределенности траектории.

>Ага, а астероид такой: "ну ок, подожду век другой, не буду въебениваться."
А вот и подождет, если при помощи такого способа можно будет точно прикладывать силу в нужном направлении. Потенциально опасные NEA покрыть и пусть поднимают орбиту себе потихоньку сами.

>Алсо, ты как красить собрался? Таджиков отправишь?
Пульверизатором распылить. Пленочкой покрыть. Не суть важно, технически это относительно реализуемо. Но даст ли это именно такой результат - тоже большой вопрос. Отклонение астеров это вообще задача выглядящая нерешаемой, если что, я не претендую на какое-то решение, просто предложил более действенный как мне кажется вариант.

>>294869
Тут фишка в том, что надо рандеву делать на встречных курсах, и определить центр масс задача нетривиальная, и точно направленный взрыв. Да и опять же, какой прок от комариного укуса.
Аноним 19/05/16 Чтв 21:54:58 #477 №294875 
>>294869
>А можно создать оболочку бомбы так, чтобы при взрыве мы получили что-то вроде плазменного копья, а не распространения во все стороны?
Нет. Да и что отбрасывать-то предлагается в этом случае, в качестве рабочего тела? Материал самой бомбуэ, разве что? Там копейки.
Аноним 19/05/16 Чтв 21:56:57 #478 №294876 
>>294873
Гигатонна не комариный укус. Если взрыв произойдет на глубине, то волны просто разорвут астероид на куски. На Земле от гигатонны произошло бы землетрясение в 10 по шкале Рихтера.

>>294875
Ну, потому и спрашиваю. Может есть какой-нибудь способ добиться хотя бы асимметрии термоядерного взрыва, чтобы энергия уходила в нужно направлении.
Аноним 19/05/16 Чтв 22:04:31 #479 №294878 
>>294876
>Если взрыв произойдет на глубине
То он произойдет на глубине. Я про поверхностный пшик.

>Может есть какой-нибудь способ добиться хотя бы асимметрии термоядерного взрыва, чтобы энергия уходила в нужно направлении.
Ну если астер предварительно хорошенько изучить, пробурить дырку в нужном месте, правильно её законопатить, то скорее всего можно.

>Гигатонна не комариный укус.
Это всего лишь 4.184х1018Дж. Даже если они все уйдут в реактивную струю с хорошей эффективностью, большой астероид так толком не подвинешь.

>то волны просто разорвут астероид на куски
Зависит от размеров, состава, свойств породы. Многие астеры состоят из дохуя льда и в перигелии проявляют кометные свойства - вот там бздыщь будет знатный, наверно.
Аноним 19/05/16 Чтв 22:45:06 #480 №294889 
14636871068470.png
Насколько велика Галактика на фоне других галактик?
Какие объекты помимо спутников и Андромеды приближаются к ней?
Аноним 19/05/16 Чтв 23:07:16 #481 №294894 
>>294627
Так то мы и так в войде.
Аноним 19/05/16 Чтв 23:12:54 #482 №294897 
>>294889
>Насколько велика Галактика на фоне других галактик?
Среди спиральных галактик абсолютно средний размер.
>Какие объекты помимо спутников и Андромеды приближаются к ней?
Галактика Треугольника
Аноним 19/05/16 Чтв 23:25:57 #483 №294903 
>>294868
Как таковых "взрывов" в космосе нету. Просто что-то врезается в ещё что-то и получается "бум", т.е. разлетаются осколки и с ещё какой-нибудь хероборой. Взрыв как на Земле, с соответствующим пламенем и всей хуйнёй так легко не получить.

>кислородосодержащие супер вещества
Ну бля, это уж совсем
Аноним 19/05/16 Чтв 23:52:45 #484 №294910 
>>294868
>Погодите, в открытом космосе могут происходить взрывы? Но как?
На химическую двухкомпонентную реакцию или тем более на ядерную/термоядерную наличие или отсутствие воздуха никак не влияет. Но воздействие взрыва не будет так эффективно передаваться, последствия гораздо меньше.
https://en.wikipedia.org/wiki/Starfish_Prime (не совсем открытый космос, околоземная орбита, но самый высокий тест)
https://www.youtube.com/watch?v=Fts8iIwn5HE
Аноним 20/05/16 Птн 01:31:26 #485 №294912 
>>294868
> Или есть какие-то кислородосодержащие супер вещества?
Есть. Порох например, прикинь.
Аноним 20/05/16 Птн 04:07:55 #486 №294916 
>>294846
>миллиард тонн ТНТ это слишком мало для придания хоть сколько-нибудь заметной скорости хоть сколько-нибудь приличному астеру
Кубического километра ВВ тебе мало? Что у тебя за астероид там, размером с Луну? Да само по себе тело в кубиком с ребром в км, может разъебать какую-нибудь австрию. Нет, ну конечно если Марс сойдет с ума и вдруг съебется с орбиты и двинется убивать всех человеков, то его только и останется, что фольгой обтягивать и молиться. Но подавляющему большинству когда либо падавших на Землю космических каменюк, хватило бы гигатонны, чтобы превратиться в радиоактивный пепел.
Аноним 20/05/16 Птн 10:45:06 #487 №294929 
>>294916
>Кубического километра ВВ тебе мало?
Речь-то о ТЯО и энергетическом эквиваленте, алё. Вперёд, доставлять миллиард тонн тротила на астер. А лучше доставь водорода и NERVA, будет гораздо эффективней, и каменюку размером с Апофис ты так все равно далеко не увезёшь. Так что да, этой энергии очень мало, по сравнению даже с гипотетической покраской.

>Да само по себе тело в кубиком с ребром в км
Вот например наиболее непосредственная угроза, недавно не очень далеко прошел от окна столкновения
https://ru.wikipedia.org/wiki/%2899942%29_Апофис
Аноним 20/05/16 Птн 10:46:14 #488 №294930 
>>294916
>подавляющему большинству когда либо падавших на Землю космических каменюк, хватило бы гигатонны, чтобы
негарантированно
>превратиться в
рой обломков, который неизвестно куда и на кого упадет.
Аноним 20/05/16 Птн 10:59:29 #489 №294933 
>>294916
Помню был какой-то фильм-катастрофа, где огромный астероид летел к земляшке. Запустили в него бомбуэ на ракете и каменюка раскололась... на два больших кусочка + мусор.
Аноним 20/05/16 Птн 11:17:27 #490 №294936 
А правда что контакт с внеземным разумом предположим он есть в принципе невозможен из-за различных путей эволюции?
Аноним 20/05/16 Птн 11:25:12 #491 №294937 
>>294930
Рой обломков не так опасен как цельная каменюка, они еще и посгорают в атмосфере, и сбивать их можно будет уже мегатонными боеголовками. Цельная каменюка может ебнуть так что разом пробудятся все спящие вулканы.
Аноним 20/05/16 Птн 11:50:25 #492 №294940 
>>294936
Наоборот, разумные формы жизни должны быть очень похожи. 2 руки, 2 ноги, стереоскопическое зрение, какое-нибудь хватательное приспособление на концах рук. Если бы чего-то из этого не было, обезьяна бы не прокачалась до человека.
Аноним 20/05/16 Птн 13:32:19 #493 №294948 
>>294940
Толсто блять ефремовец
Аноним 20/05/16 Птн 14:15:14 #494 №294952 
>>294912
>Порох
А как ты его воспламенишь, ало?
Аноним 20/05/16 Птн 14:41:52 #495 №294953 
>>294910
А атмосфера влияет на распространение электромагнитного импульса и излучения?
Аноним 20/05/16 Птн 14:46:29 #496 №294954 
Откуда астероиды получают ускорение? гравитационное ускорение?
Аноним 20/05/16 Птн 15:01:02 #497 №294955 
>>294707
Часть оболочки, которая рассеется дальше орбиты Юпа, уже не будет притягивать его к центру масс.
Ваш К.О.
Аноним 20/05/16 Птн 15:31:46 #498 №294957 
14637475066900.jpg
>>294894
Кто в войде, ты в войде, епт. Мы на окраине филамента, да, но нихуя не в войде.
Аноним 20/05/16 Птн 15:53:22 #499 №294959 
>>294952
Слишком тупо даже для этого треда. точно так же как и в атмосфере

>>294954
Нихуя не понял о чем ты, поясни. Если ты об ускорении свободного падения, то от гравитации других тел.
Аноним 20/05/16 Птн 16:03:01 #500 №294961 
14637493816340.jpg
А что произойдёт с планетой поглащённой красным гигантом в следствии расширения этого гиганта? Есть ли шанс что построив ёба холодильник можно пережить под поверхностью или она будет падать к ядру?
Аноним 20/05/16 Птн 16:05:29 #501 №294962 
>>294953
Да, например гамма и рентген почти не проникают через атмосферу. УФ и микроволны тоже частично блокируются на определённых длинах волн. Некоторые длины радиоволн отражаются от ионосферы. Видимый свет так же переизлучается, поэтому небо голубое.

Аноним 20/05/16 Птн 16:09:08 #502 №294964 
>>294961
Сдерет атмосферу, потом расплавит кору и мантию, все это так же сдует. Останется ядро. Если будет полное погружение, то планета расплавится полностью и распадётся на мелкие писюльки и на атомы.
Аноним 20/05/16 Птн 16:09:53 #503 №294966 
>>294959
>от гравитации других тел
Я об этом
Аноним 20/05/16 Птн 16:44:17 #504 №294969 
>>294961
Холодильник нет смысла строить, есть смысл либо увеличить орбиту, если доступна такая возможность, либо перебираться на Плутон, который к тому времени как раз попадет в обитаемую зону. Да и вообще еще раньше надо смотреть, куда съябывать, поскольку возле получившегося в итоге белого карлика сильно не разживешься.
Аноним 20/05/16 Птн 17:25:04 #505 №294973 
>>294969
К тому времени, как солнышко начнёт раздуваться, то человечества либо уже не будет, либо оно сможет построить что-нибудь огромное и автономное и съебёт из солнечной системки навстречу приключениям.
Но это уже сай-фай.
Аноним 20/05/16 Птн 17:45:27 #506 №294975 
>>294969
Плутон не попадёт. Но на Титане будет неплохо.
Аноним 20/05/16 Птн 18:06:39 #507 №294978 
>>294948
Хуле толсто. Только у всяких недалёких фантастов, рассчитывающих удивить/испугать читателя, встречается разум со щупальцами, в виде океана или вообще какого-нибудь электричества. Когда к человеку наконец прилетят Вуглускры с планеты Жопа, тогда ты поймёшь, чего стоит твой биологический романтизм. Скринь пост кароч.
Аноним 20/05/16 Птн 18:06:47 #508 №294979 
>>294969
Солнце уже через 500млн лет увеличит яркость почти в два раза, т.е еще за долго до красных гигантов Земля станет второй Венерой
Аноним 20/05/16 Птн 18:08:57 #509 №294980 
>>294964
Так будет погружение в слои ниже?
Просто если взять ёба магнитное поле и окружить им планету, то наверняка верхняя фотосфера, которая раздуется на столько что станет жиже в охулиард раз, не сможет это магнитное поле распидорасить.
Аноним 20/05/16 Птн 18:10:17 #510 №294981 
>>294979
А твари божьи успеют приспособиться?
Аноним 20/05/16 Птн 18:28:22 #511 №294988 
>>294978
Люди развились до этого уровня не потому что у нас есть руки ноги и 2 глаза, а потому что им повезло. 2 млн лет от тупой обезьяны это пиздец как быстро. Всякие маня теории о руках и труде лишь подгонка под ответ. Если взять других умных животных использующих инструменты, ты обнаружишь что вороны, осьминоги, дельфины и слоны, ты обнаружишь, что ничего напоминающего человеческую руку у них нет. А у некоторых и глаз.
Даже если углеродный шовинизм, то не факт что разумная форма жизни не будет какая-нибудь прыгающая улитка с гибкими челюстями.
Аноним 20/05/16 Птн 19:07:55 #512 №294993 
>>294954
Откуда и планеты. Все вещество когда-то крутилось в составе протопланетных дисков. Это остатки роскоши былого момента подаренного звездой.
Аноним 20/05/16 Птн 20:31:58 #513 №295003 
14637655187390.jpg
>>294962
> гамма и рентген почти не проникают через атмосферу
Аноним 20/05/16 Птн 20:33:54 #514 №295006 
>>294980
тормозить будет землю, которая будет понижать орбиту пока не ебнется в центр
Аноним 20/05/16 Птн 20:35:57 #515 №295008 
>>294966
У них нет ускорения. Они падают в невесомости
Аноним 20/05/16 Птн 20:37:24 #516 №295009 
>>294969
> возле получившегося в итоге белого карлика
до белого карлана надо дожить еще, ибо новая звезда плюнет протопланетным облаком при его образовании
Аноним 20/05/16 Птн 20:41:05 #517 №295011 
>>294980
> ёба магнитное поле и окружить им планету
> фотосфера, которая раздуется
> не сможет это магнитное поле распидорасить
Магнитное поле материально. Если пнуть магнитную подушку, она передаст энергию, и земля будет так же терять орбитальную скорость как и без неё, разве что поверхность не спалит
Аноним 20/05/16 Птн 20:44:44 #518 №295013 
>>295003
Представь себе, ньюфажик. Именно поэтому рентген и гамма-телескопы запускают на орбиту, а не строят на земле.
Аноним 20/05/16 Птн 20:51:07 #519 №295015 
>>294979
Через 3,5 млрд на 40%.
> A luminosity of 1.1 Lsun will be reached in 1.1 Gyr, and 1.4 Lsun in 3.5 Gyr
Аноним 20/05/16 Птн 20:51:55 #520 №295016 
>>294897
Насколько абсолютно? Может ли быть так же как и с красными карликами, вроде того же треугольника, что видим далеко только большие?
Аноним 20/05/16 Птн 21:11:07 #521 №295020 
>>295016
>Насколько абсолютно?
Сильно абсолютно.
>Может ли быть так же как и с красными карликами, вроде того же треугольника, что видим далеко только большие?
Что блядь?
Аноним 20/05/16 Птн 21:35:58 #522 №295026 
>>295020
Вот есть красные карлики которых много в Галактике. Их больше чем всех других звёзд. Но видны только ближайшие. Может ли так обстоять дела и с галактиками на подобие треугольника?
Аноним 20/05/16 Птн 21:42:52 #523 №295030 
Если на белый карлик вылить немного вещества недостаточного для ядерных или гравитационных реакций будет ли от столь же белым или он потускнеет хоть чуть-чуть?
Аноним 20/05/16 Птн 21:56:05 #524 №295035 
Если белый карлик попадёт в газопылевую область или остаток сверхновой загорится ли она вновь, учитывая что это одиночный объект?.
Аноним 20/05/16 Птн 23:05:39 #525 №295049 
>>294988
Никакая это не подгонка. У осьминогов, дельфинов и воронов (лолшто?) было гораздо больше времени занять человеческую нишу, но только обезьянообразное существо смогло это сделать. Инструменты могут использовать даже насекомые, но дело не в самих инструментах, а в универсализме. Нет в природе органа более универсального, чем человеческая (обезьянья) рука. И это совсем не случайность, что именно такая форма тела пришла к успеху. Да хуле говорить. Даже шимпанзе из-за немного отличающегося расположения большого пальца не способно к настолько тонкой моторике, как рука Homo Sapiens. Проще говоря, ты не сможешь построить космическую вундервафлю плавниками или гибкими челюстями.
Аноним 20/05/16 Птн 23:20:25 #526 №295050 
>>295013
Радиационный фон настолько мал, что на земле будут помехи от некосмических источников, нюфажик
Аноним 20/05/16 Птн 23:24:30 #527 №295051 
>>295026
галактики достаточно большие чтобы их видеть. Но кроме них есть еще всякие облака типа магелланова, шаровые скопления и прочая чухонь. Сколько её - хуй знает.
Аноним 20/05/16 Птн 23:25:41 #528 №295052 
>>295030
гравитационное сжатие жи, побелеет сильнее
Аноним 20/05/16 Птн 23:26:14 #529 №295053 
>>295035
засосёт материи, перейдет предел чандрасекара и уебет сверхновой
Аноним 21/05/16 Суб 00:45:02 #530 №295068 
>>295035
Раздует пыль излучением и/или ветром, и нихуя не наглотается.
Аноним 21/05/16 Суб 01:25:33 #531 №295070 
Можно ли сделать интенсивные магнитные поля крутой мощности но не разлетающиеся на хулиард св.лет?
Аноним 21/05/16 Суб 01:57:49 #532 №295075 
14637850693470.gif
>>295050
Шел бы ты нахуй, петушок.
Аноним 21/05/16 Суб 01:59:36 #533 №295076 
>>295070
Да, можно заебенить магнетар, например.
Аноним 21/05/16 Суб 02:05:54 #534 №295077 
>>295035
>в газопылевую область
Смотря какая насколько плотная область.
>остаток сверхновой
Разлетается слишком быстро, вещества недостаточно
>>295026
Не может там быть слишком много красных карликов. В Галактике Теругольника ясно читаются спиральные рукава, зоны активного звездообразования, населенные молодыми и массивными звездами.
Аноним 21/05/16 Суб 09:17:21 #535 №295083 
14638114418540.jpg
>>295049
> строить вундервафлю
> руками
Аноним 21/05/16 Суб 11:48:00 #536 №295092 
>>295068
Не неси хуйню, для того, чтоб что-то сдувать, излучение должно быть больше эдингтоновского предела, который для карлика солнечной массы равен 32000 светимостей солнца, тогда как у карлана оно в миллионы раз слабее.
>>295053
Собственно, до чандрасекара масса немного не дойдет, потому что ядро нагреется и бомбанет раньше. А так да, получим сверхновую тип Ia.
Аноним 21/05/16 Суб 13:16:51 #537 №295113 
>>295049
Выгодность руки не в том что там 5 пальцев, а в том что их две. У осьминога их 8 и единственная причина почему они не пронзили небеса это то что у них не было охуевшего количества совпадений. И не стоит забывать о том что основной причиной был мозг, а не руки.
И нет ни одной причины почему разумная форма жизни не может выглядеть как какой-нибудь носорог с комариными ногами головой на жопе и осминожьими щупальцами на спине. В конце концов это может быть просто высокоорганизованная колония микроорганизмов.
Аноним 21/05/16 Суб 13:39:36 #538 №295114 
>>295113
Удваиваю этого ксенобиолога
Аноним 21/05/16 Суб 16:42:10 #539 №295126 
>>295113
>основной причиной был мозг, а не руки.
А что если мозг и развился от рук?
Аноним 21/05/16 Суб 17:01:34 #540 №295129 
14638392944470.jpg
>>295126
Мозг появился благодаря хуевой тучи факторов как внешних, так и внутренних. Да, частично руки способствовали. Но это не значит, что не будь рук не было бы и мозга. К тому же когда говорят "рука" имеют ввиду кисть способную что то схватить Большой палец конечно удобная фича но не решающая, точно в таком же ключе можно сказать и о осьминожьей руке.
Аноним 21/05/16 Суб 19:28:08 #541 №295134 
>>295113
Сначала пишешь, что причина в количестве конечностей, и сразу же оправдываешь неразвитость осьминогов тем, что им просто не повезло. Да и билатеральная симметрия вообще самая распространённая на Земле. Чёт кроме человека никто к успеху не пришёл.
>У осьминога их 8 и единственная причина почему они не пронзили небеса это то что у них не было охуевшего количества совпадений.
>Выгодность руки не в том что там 5 пальцев, а в том что их две.
Хуйню сказал. Всё дело в мелкой моторике, остальное несущественно. То есть кисть и расположение большого пальца определяющие факторы. Как ты будешь делать микроскопы, телескопы и транзисторы щупальцами? Разум сам по себе нихуя не значит, если он не способен понять себя и покинуть планету где он появился.
Все эти твои "разумные" парящие слизни на пердяной тяге останутся на уровне наших осьминогов или дельфинов. Говно это, а не разум.
>И нет ни одной причины почему разумная форма жизни не может выглядеть как какой-нибудь носорог
Тысячи причин.
> В конце концов это может быть просто высокоорганизованная колония микроорганизмов.
Просто фейспалм.
Аноним 21/05/16 Суб 19:35:48 #542 №295135 
>>295134
У нас тут гуманоидный шовинист завелся
Аноним 21/05/16 Суб 19:43:20 #543 №295136 
>>294957
Это лишь теоритическая матмодель. Лысые макаки еще слишком тупы, чтобы заглядывать так далеко.
Аноним 21/05/16 Суб 20:13:16 #544 №295139 
>>295134
А что если люди это только первые организмы с разумом? Как первые позвоночные или первые сухопутные и через пару десятков миллионов лет на Земле большинство существ будут разумными
Аноним 21/05/16 Суб 20:33:01 #545 №295141 
>>295139
Для этого нужны УНИКАЛЬНЫЕ условия, что было на земле единожды и не будет уже никогда.
Аноним 21/05/16 Суб 20:47:17 #546 №295142 
>>295141
>не будет уже никогда
Ванга?
Аноним 21/05/16 Суб 20:51:54 #547 №295143 
>>295142
Прочитавший учебник биологии
Аноним 21/05/16 Суб 20:52:44 #548 №295144 
>>295142
На самом деле немного погорячился, но в целом это пиздцки маловероятно
295143 - кун
Аноним 21/05/16 Суб 20:52:54 #549 №295145 
>>295141
>бескнонечная галактика
>не будет никогда
Аноним 21/05/16 Суб 20:56:03 #550 №295146 
>>295145
>нихуя не читаю
>отвечаю
Аноним 21/05/16 Суб 21:15:17 #551 №295151 
14638545174280.gif
>>295145
>бескнонечная галактика
Аноним 21/05/16 Суб 21:54:27 #552 №295153 
>>295134
Я писал, что дело не в конечностях. У многих есть хватательная хуйня. Большой палец хоть и помогает, но не решает. Убери палец, прогресс хоть и замедлится, но не остановится. А осминогам не повезло ввиду того же, почему не повезло любому другому животному кроме человека.
> микроскопы, телескопы и транзисторы щупальцами?
Да, почему нет? У них охуенные присоски, лучше чем у человека. Хобот тоже самое, немного развить конец и вот тебе ёба манипулятор.
>Тысячи причин.
Назови их.
>Просто фейспалм.
Антропоцентризм. Есть все шансы на это.
Аноним 21/05/16 Суб 22:05:09 #553 №295154 
>>295141
Ну вот скажем. Через н-сот млн. лет солнце начнёт греть . Люди скорее всего съебут, кто на планету железяка, кто в бункеры с клонами. Живность начнёт привыкать к жаре. Люди хоть и могут привыкнуть тоже, но лишь искусственно и не будет иметь массовый характер. Будут дикие земли. + Звери вероятно понаедут в другие системы как объекты тераформинга. В каких то системах люди могут все подохнуть или скатится в технофентези. И спустя н лет могут эволюционировать.
Аноним 22/05/16 Вск 00:03:00 #554 №295164 
Зачем нужен стартовый стол?
Газоотвод понятное дело, бетонка все дела, ж\д, аэропорт. Но в чём сложность?
Одно дело попилы, оно понятно. Пускай реальная цена в 10 раз меньше. Но это же всё равно много.
Аноним 22/05/16 Вск 00:13:02 #555 №295166 
>>295164
Фундамент, точка опоры, защита для коммуникаций.
Цену посчитать попроси у тех кто строит такие объекты. Материалы там не проходные, как и технология и требования. В теории естественно.
Аноним 22/05/16 Вск 01:10:17 #556 №295167 
Почитал про тепловую смерть Вселенной и что-то подохуел. Ну, ладно Земли не станет, ладно, нашей галактики, но представьте, что не станет всей Вселенной. Просто представьте на секунду, что Вселенная ВСЁ. То есть если нет Земли - есть ещё куча других планет, если нет Млечного Пути - ещё дохуя галактик. А так нет НИЧЕГО, бытие кончилось. Мы не знаем, есть ли другие вселенные, представьте, что нет. Меня уже который день морозит от этой мысли. НИЧЕГО не будет, но самое страшное, что не будет НИКОГО кто бы это осознал. Заместо нашей уютной вселленной будет какая-то ебучая непозноваемая пустота. Всё, что происходит сейчас или происходило когда-либо не будет иметь никакого смысла. И НИКТО не сможет об этом даже подумать, потому что этого кого просто не будет существовать. Это просто самая страшная вещь, которую я когда либо осознавал. Пишу это, и меня слегка колотит. Все же надеюсь, что другие вселенные есть, и со смертью нашей не настанет всеобщий хтонический конец ВСЕГО.
Аноним 22/05/16 Вск 01:13:50 #557 №295168 
>>295167
Мне похуй
и немедленно накатил
Аноним 22/05/16 Вск 01:15:46 #558 №295169 
>>295166
Маск строит космодром за 100млн баксов, а Восточный уже почти 3млрд, там по слухам только один подрядчик спиздил 100млн баксов. И только два пуска в следующем году будет т.е. он походу и не нужен никому оказался, а у Маска же космодром будет как конвейер запускать ракеты.
Аноним 22/05/16 Вск 02:49:51 #559 №295175 
>>295136
А это шо за хуйня ещё?
>По спорным утверждениям некоторых физиков, реликтовое холодное пятно может быть отпечатком другой вселенной за пределами нашей
Аноним 22/05/16 Вск 03:15:17 #560 №295177 
>>295175
https://www.newscientist.com/article/mg19626311.400-the-void-imprint-of-another-universe/

Ну там типа, ну прилипли двумя вселенными, как два мыльных пузыря жопами, оно и отпечаталось.
Аноним 22/05/16 Вск 12:42:59 #561 №295190 
>>295167
Нет это не страшно. Всё имеет свой конец. Если существует 1 вселенная, то нет никаких ограничений на существование ещё 1. Другой вопрос о смерти мультивселенной. Однако если представить эти вселенные как некие самоподдерживаемые флуктуации, то твой пост окрашивается уже в другой оттенок.
Аноним 22/05/16 Вск 14:14:38 #562 №295194 
14639156782210.png
>>295169
На самом деле куда больше.
Аноним 22/05/16 Вск 15:28:57 #563 №295201 
А что если Великий Аттрактор – это разрыв пространства?
Аноним 22/05/16 Вск 16:37:12 #564 №295206 
>>295201
А что если нет?
ПЕРЕКОТ Аноним 22/05/16 Вск 16:50:45 #565 №295208 
Перекатываемся, уважаемые анонимные астрофизики.
>>295207 (OP)
https://2ch.hk/spc/res/295207.html
Аноним 22/05/16 Вск 16:52:48 #566 №295209 
14639251682890.png
>>295208
>астрофизики
Аноним 22/05/16 Вск 19:06:35 #567 №295217 
14639331951780.jpg
>>295167
Читай про мультиверс и бесконечную инфляцию, которая постоянно порождает новые вселенные с рандомными параметрами физики из квантовых флуктуаций.
Аноним 24/05/16 Втр 17:32:59 #568 №295376 
>>294878
> состава, свойств породы
чем плотнее, тем больше вероятность растроньшыть его в песочек.

олсо бомбить можно не обязательно на подлёте, а за 2-3 оборота до. Красить же нужно за сотню лет до импакта.

олсо олсо астеры до километра еще можно поправить, а всякое крупное говно типа весты не забонбить ничем
Аноним 25/05/16 Срд 09:03:41 #569 №295403 
>>292346
нисколько
гугли гравитационный потенциал
Аноним 25/05/16 Срд 11:15:55 #570 №295408 
>>295167
Про антропный принцип теперь прочитай. Алсо времени не существует. Мы видим эволюцию вселенной последовательно только потому, что в ней находимся и работает по её физическим законам.
Аноним 28/05/16 Суб 11:43:47 #571 №296143 
А вот если , положим, земле грозит апокалипсиснейм и через лет 5 разом выпилит всю жизнь. Все страны решают объединить усилия, ресурсы и ,построив корабли, свалить отсюда. При каком технологическом развитии это возможно?
Аноним 14/06/16 Втр 22:52:57 #572 №299905 
>>296143
Пиздец всем, как не ебись. Если хочешь поговорить об этом с фантастической копипастой, то есть такой канадский фантаст - Нил Стивенсон, а у него есть книжка - Sevenevs. Уровень развития прекрасное завтра в космосе только допиленная МКС с пристыкованным астероидом, все летают на союзах, а полный пиздец наступает через два года, человечество пытается засунуть всех кого только можно на орбиту.
Аноним 22/06/16 Срд 20:29:12 #573 №301406 
а что если нас наебали и к марсу ни один зонд хуёнд, марсоход хуеход, вовсе не летали, что если NASA показывает нам фотки какой-нибудь мексики)))
Аноним 28/06/16 Втр 00:41:28 #574 №302164 
>>292584
Почитай законы Ньютона, затем приходи.
Аноним 28/06/16 Втр 00:57:22 #575 №302166 
>>293926
>Что может сделать человек чего не сможет сделать робот?
Crew report.
comments powered by Disqus

Отзывы и предложения